Sunteți pe pagina 1din 82

Назва наукового напрямку (модуля): Семестр: 12

Infectious diseases situational tasks


Опис:
med. 6 course
Перелік питань:
1. ?Patient G., 41 years old, has high temperature during 8 days, severe headache,
constipation. Objectively: body temperature is 39,5 °C, pale, languid. Pulse 82
per min, a tongue is dry, covered by the brown coat. An abdomen is moderately
exaggerated, painful in a right ileac area. A liver + 2 cm. What is the most
A. * credible fever
Typhoid diagnosis?
B. Epidemic typhus
C. Flu
D. Appendicitis
E. Yersiniosis
2. Patient L., 20 years old, had appendectomy on the 5th day of disease. During
operation was found an appendix with the signs of catarrhal inflammation and
hyperplasia of lymphatic nodes of mesentery. After a day his state became
severe: temperature – 40 °C, with hallucinations, hepatosplenomegaly. Single
roseolar elements appeared on the skin of abdominal on a 10th day. The tongue
is great, covered with grey coating, with the imprints of teeth, operation wound
A. * Typhoid fever, atypical form: appendicotyphoid
B. Epidemic typhus
C. Yersiniosis
D. Pseudotuberculosis
E. Flu
3. Patient L., 20 years old, had appendectomy on the 5th day of disease. During
operation was found an appendix with the signs of catarrhal inflammation and
hyperplasia of lymphatic nodes of mesentery. After a day his state became
severe: temperature – 40 °C, with hallucinations, hepatosplenomegaly. Single
roseolar elements appeared on the skin of abdominal on a 10th day. The tongue
is great, covered with grey coating, with the imprints of teeth, operation wound
A. Urinoculture
B. Coproculture
C. Biliculture
D. * Blood culture
E. Reaction of Vidal
4. Pain in abdomen appeared at a patient with typhoid fever on the 19th day of
disease, and last 4 hours. Pulse – 100 per min, rhythmic. A tongue is dry,
covered by white coat. The abdomen is tense, does not take part in the act of
breathing. Stool and urine are absent. What complication about is it necessary to
A. think?
Appendicitis
B. Infectiously-toxic shock
C. Bleeding
D. * Perforation
E. Nephrolithiasis
5. At the patient B., 25 years old, was diagnosed typhoid fever. On the 17th day of
disease the body temperature critically decreased to the norm, a pale color of
skin progress. Consciousness is saved. Pulse 120 per min, rhythmic. On the top
of heart is systolic noise. Constipation. What complication about is it necessary
A. * to think?
Bleeding
B. Perforation
C. Infectious-toxic shock
D. Infectious-allergic myocarditis
E. Pneumonia
6. At the typical form of typhoid fever temperature of body rises like stairs day by
day to 39-40 °C at the end of 1st week, and then slowly during 2-3 week
gradually decreases, comes to the norm. What is the name of such temperature
A. curve?
Vunderlih’s temperature curve
B. Botkin’s temperature curve
C. * Kildyushevsky’s temperature curve
D. Eller’s temperature curve
E. Intermedium temperature curve
7. At the typical form of typhoid fever temperature of body rises like stairs day by
day to 39-40 °C at the end of 1st week, than during 10-14 days stays
approximately on this level, and then becomes remittent and, gradually
A. * decreases, comes
Vunderlih’s to the curve
temperature norm. What is the name of such temperature curve?
B. Botkin’s temperature curve
C. Kildushevsky’s temperature curve
D. Eller’s temperature curve
E. Intermedium temperature curve
8. Sick person, 23 years old, appealed to the hospital on the 6th day with gradual
development of illness, with complaints of severe headache, pain in the back of
head, violation of sleep, fever. Objectively: Ò-39,7 °C, Ðs-84/min. A face is pale,
tongue dry, near a root covered by the grey covering. A stomach is flatulent.
During percussion the sound is dull in a right iliac area. A liver and spleen are
A. On 12 day
B. On 4 days
C. On 5 day
D. On 6 day
E. * On 8 day
9. One of the methods of diagnostics of typhoid fever and paratyphoid fever is
hemoculture. For that in a fever period inoculation of blood from a vein on
bilious broth or Rappaport‘s medium in correlation 1:10 is made. What volume of
A. blood
5 ml ofmust be taken on the 2nd week of disease?
blood
B. 10 ml of blood
C. * 15 ml of blood
D. 20 ml of blood
E. 25 ml of blood
10. To the admitting room of infectious hospital came patient with complains of high
fever – 38-40 °C during 3 weeks, headache, weakness, insomnia. She didn’t
consult with the doctor. She took antipyretic drugs, seduxen. Objectively: Ò 35,7
°C, Ðs – 140/min., BP-80/50 mm Hg. General condition is severe. Skin and
mucous membranes are pale. The tongue is thickened with the imprints of teeth,
with the dirtily-brown covering, apex and edges of tongue clean. A stomach is
flatulent. A liver and spleen is moderately enlarged. There was a stool on the
A. Drugs poisoning
B. Infectiously-toxic shock
C. Hemorrhoid bleeding
D. * Intestinal bleeding
E. Perforation of colon

11. Patient Í., 28 years old, entered to the clinic on the ninth day of illness with
complaints of the increased Ò to 39,0 °C, headache, general weakness,
constipation, violation of sleep. At a review: on the skin of abdomen are single
roseollas, a tongue is covered by the brown covering, Ps 78 per min, rhythmic, a
A. liver is enlarged
Bradycardia, on 2ñm.
dicrotic What
pulse, changes
muffling will be tones,
of cardiac from the side of the cardio-
hypotension
B. Tachycardia, dicrotic pulse, muffling of cardiac tones, hypotension
C. Tachycardia, dicrotic pulse, muffling of cardiac tones, hypertension
D. * Relative bradycardia, dicrotic pulse, muffling of cardiac tones, hypotension
E. Dicrotic pulse, muffling of cardiac tones, hypotension
12. A 30 y/o patient is seen on the 9th day of illness. The symptoms of illness had
been building-up gradually with increasing of fever and intoxication. Roseolar
rash has appeared on skin of abdomen. The skin is pale, temperature is 40 °C,
pulse 80/per 1 min., BP 100/65. The tongue is covered with sediments and
abdomen is swollen. Spleen and liver are palpable. What symptom will be
A. Symptom of Botkin
B. * Symptom of Padalka
C. Symptom of Kildyushevsky
D. Symptom of Ortner
E. Symptom of Pasternatsky
13. A sick woman, 32 years, complains on diarrhea, headache, severe weakness,
insomnia, dull pain in a right iliac area. It is a 8th day of illness. At a review: Ò-
39,8 °C, Ðs – 86/min., BP – 90/60 mm Hg. Dicrotic pulse. Skin is pale. Single
roseollas are on the abdomen. Tongue is dry, covered by the coverings, with the
imprints of teeth on a lateral surface. Soft, dulling of percussion sound is
determined in an illeocaecal area of abdomen. Hepatosplenomegaly, positive
A. No changes
B. * Presence of air under a diaphragm
C. Presence of the exaggerated loops of intestine
D. Enlarged liver and spleen
E. Signs of impassability of intestine
14. Patient C., 35 years old, the disease began with severe chills, raising the
temperature to 39 ?C, vomiting, pain in epigastric region, diarrhea with water-
smelly bowel movement. Over 6 hours after eating raw eggs, potatoes with
A. * braised meat, drank juice. What kind of agent is likely to cause this condition.
Salmonella
B. Vibrio cholera
C. Enteric stick
D. Campylobacter
E. Shigella
15. Patient C., 22 years old, hospitalized in an infectious department complaining of
chills, increasing of temperature to 38,5 ?C, vomiting, pain in epigastrium,
diarrhea. 7 hours before the disease, ate raw eggs, potatoes with braised meat,
drink tomato juice. What kind of organism most likely to cause disease.
A. Enterovirus
B. * Salmonella
C. Enteric stick
D. Staphylococcus
E. C. perfringens
16. Emergency ambulance delivered in infectious hospital girl N., 17 years old,
complaining of headache, expressed general weakness, pain in epigastrium,
repeated vomiting, greenish color diarrhea to 8 times a day. She ate 2 raw
chicken eggs. Body temperature is 39 °C, the tongue covered with white coat,
A. moderately
Acute cramps, pain in abdomen and sigmoid area. The most likely
appendicitis
B. Crohn’s disease
C. Shigellosis
D. Viral gastroenteritis
E. * Salmonellosis
17. Patient F., 25 years old, was hospitalized in the infectious hospital with
complains on frequent vomiting, liquid stool, abdominal pain in epigastrium. Pale
skin, dry tongue with a grey cover, hypotension, tachycardia. The doctor
A. suspected salmonella.
Microprecipitation What serologic study is needed to perform.
reaction
B. RA (Widal)
C. * RIGA with Salmonella diagnosticum with paired serum
D. RIGA with Shigella diagnosticum and RA (Widal) with paired serum
E. RKC
18. Patient L., 32 years old, was admitted to the infectious hospital, complaining of
chills, headache, expressed general weakness, cutting pain in epigastrium,
diarrhea. The temperature is 39 °C, repeated vomiting. Excrement abundant,
greenish color. Over 4 hours to illnesses ate the meat salad in the factory
A. canteen. Which
Antibotulism drugs will be the most effective for the treatment of this patient.
serum
B. Salt and colloid solutions
C. Sulfanilamides
D. * Antibiotics
E. Enterosorbent
19. Patient B., 55 years old, was hospitalized in an infectious hospital with complains
on frequent vomiting, liquid excrements, abdominal pain, mainly in the right iliac
area and epigastrium. Pale skin, dry tongue with gray covering, hypotension,
tachycardia. The doctor suspected salmonella. What laboratory test is need to
A. perform. blood analysis
Common
B. Parasytoscopy of blood smear
C. Microscopy of stool
D. * Coproculture
E. Biologic test
20. Patient C., 30 years old, was hospitalized with a preliminary diagnosis of
salmonellosis. Body temperature – 40 °C, repeated vomiting, profused diarrhea.
Hypotension (BP 55/30 mm Hg), expressed tachycardia (pulse to 120/min), face
pale, pain mainly in right iliac area and epigastrium, hepatosplenomegaly were
A. found. What treatment
Glucocorticoids, tactic
crystalloid will be most
solutions, effective
antibiotics, in this case.
diuretics
B. Cardiac glycosides, colloid solutions, antibiotics
C. Sorbents, diuretics, desintoxication
D. Dopamine, colloid solutions, antibacterial
E. * Glucocorticoids, crystalloid solutions, antibiotics
21. Patient N., 22 years old, was admitted to the infectious hospital complaining on
chills, headache, expressed general weakness, cutting pain in sigmoid area,
diarrhea. The temperature of the body 39,3 °C, vomiting. Feces are watery.
Patient ate the salad with sausage and eggs. For diagnosis doctor prescribed
RIGA with salmonella diagnostic. When is it necessary to take blood for the
A. At the 1st and 2nd days of illness
B. Once in recovering period

C. * At the end of the 1st week and after 7-10 days


D. At the end of the 1st week and after 3-4 days
E. Once in acute period
22. Patient B., 32 years old, an employ of poultry was hospitalized with complains of
pain in abdomen, mostly in sigmoid area, fever up to 38,8 ?C, nausea, vomiting,
diarrhea with dark-green colour of stool, without any pathological changes. Skin
and mucous membranes are dry, moderate tension in epigastric region and pain
A. in the appendicitis
Acute right iliac area. The most likely diagnosis.
B. Cholera
C. Disbacteriosis
D. * Salmonellosis
E. Rotavirus gastroenteritis
23. 3 patients were delivered with symptoms of fever, diarrhea and vomiting.
Excrements are dark-green, without any pathological changes. All patients
together took part in the preparation of food and ate eggs, meat salad. The most
A. likely diagnosis.
Cholera
B. Botulism
C. Dysbacteriosis
D. * Salmonella
E. Shigellosis
24. Patient L., 33 years old, admitted to the hospital with the complains of
intolerable pain in the abdomen, mostly in sigmoid region, fever up to 38,8 ?C,
nausea, vomiting, diarrhea of dark-green colour in every 1-1,5 hour, without
admixture of blood and mucus. The condition is progressively deteriorated,
increasing hypotension, paleness, weak cardiac sounds, decreased intestinal
A. Glucocorticoids, infusion of salt solutions, antibiotics intravenously, diuretic
B. Cardiac glycosides, infusion colloidal solution, antibacterial drugs
C. Enterosorbents, diuretic, desintoxication therapy
D. Mezaton 1% in drops, infusion of salt solutions, antibacterial drugs orally
E. * Glucocorticoids, infusion of salt solutions, antibiotics intravenously
25. Patient 45 years old, became ill within 10 hours after consumption of grinded
meat. He complains on vomiting, pain in epigastric region, diarrhea of green
color, increasing of temperature up to 39 °C. During objective examination
revealed that: patients general condition is severe, pain in the muscles of the
lower limbs, pale face, acrocyanosis. Pulse 130 per min. BP is 70/40 mm Hg.
weak cardiac sounds, dry tongue, brown-white covering, abdomen cramps,
A. Shigellosis
B. Cholera
C. * Salmonellosis
D. Viral gastroenteritis
E. Eshericiosis
26. Patient G., 24 years old, at 6 am after taking his breakfast with boiled eggs and
a cup of coffee, filled chill, fever up to 38,8 ?C, pain in left inguinal area, nausea,
vomiting, rumbling in the stomach, then frequent diarrhea, feces copious,
A. watery, muddy-green. What is the most likely diagnosis.
Cholera
B. Botulism
C. * Salmonellosis
D. Enteric typhus
E. Shigellosis

27. Patient U., who arrived from the Crimea, at 5 o'clock in the morning appeared
diarrhea in an interval of 1-1.5 hour, with copious watery excrements, without
admixtures of mucous or blood. He was admitted in an infectious department
with the diagnosis of "acute intestinal infection". What laboratory tests are
A. necessary
General to confirm
blood for the etiological diagnosis?
analysis
B. Bacteriological blood analysis
C. Microscopy of stool
D. * Coprogram
E. The biological test in mice
28. Patient C. with the complains of diarrhea with mucus, stabbing pain in epigastric
area, rumbling in the stomach, fever. According to his anamnesis he took
preserved milk, eggs, salad of fresh vegetables. What is the most likely disease?
A. Shigellosis
B. Yersiniosis
C. * Salmonellosis
D. Cholera
E. Botulism
29. Patient B., 38 years old, came to the admission department with the complains
of fever up to 38 °C, vomiting 4 times a day, diarrhea up to 6 times in a day.
During objective examination revealed that BP is 125/75 mm Hg, pulse 80 per
min, abdomen is soft, painful in the right iliac and epigastric area .According to
anamnesis he took salad of boiled eggs and sausage. What treatment should be
A. Antibotulinum serum
B. Intravenous antibiotics
C. Washing of the stomach and intestine, rehydration therapy , glucocorticoids
D. * Gastric lavage and washing of intestine, rehydraton therapy, enterosorbents
E. Treatment after getting of the laboratory test results
30. Patient O., 27 years old, complains of headache, weakness, pain in epigastric
area, vomiting, diarrhea (9 times in a day; huge excrement of greenish colour).
Before the appearance of disease he ate raw eggs. Objectively: body
temperature is 38,8 °C. Tongue is white, pain in epigastric and umbilical region.
A. * The most likely diagnosis.
Salmonellosis
B. Cholera
C. Dysentery
D. Viral gastroenteritis
E. Acute appendicitis
31. Patient K., 30 years old, came with complains of nausea, vomiting, pain in
epigastrium and paraumbilical area. High body temperature, pale skin, dry
tongue covered with whitish-gray coating, quick pulse, low blood pressure, 2
hours before the onset of symptoms of the disease he ate in cafe. What is the
A. most likely diagnosis.
Cholera
B. Shigellosis
C. Typhoid fever
D. * Salmonellosis
E. Meningitis
32. Patient N., 30 years old, works in a restaurant, admitted to the hospital
complaining of unbearable pain in the abdomen, increasing of temperature to
38,8 ?C, nausea, vomiting, diarrhea of dark-green colour. He took eggs,
mushrooms, canned food. Objective state: general condition is moderately
A. severe, skin and mucous membranes are dry, pain in epigastric region. Name
Shigellosis

B. Yersiniosis
C. * Salmonellosis
D. Cholera
E. Rotaviral infection
33. Patient 35 years old, with complains on increasing the temperature to 39 °C with
chill, vomiting, pain in epigastric region, diarrhea with foul smell, abdominal
cramps. 6 hours before onset of the disease ate raw eggs, fried potatoes with
grinded meat, drank juice. To clarify the diagnosis from serological diagnostic
methods are often used RIGA with the group of salmonella diagnostic and RA
A. In the first day of illness
B. At the end of the first month
C. In 1st week in 3-4 days
D. * At the end of the 1st week and after 7-10 days
E. During admission to the hospital
34. A patient was hospitalized after 4 hours of consumption of raw eggs,
complaining on frequent vomiting, abdominal pain, mostly in the right iliac area
and umbilical area. Objectively: pale skin, dry tongue with gray coating,
A. hypotension,
General bloodtachycardia.
analysis What tests should be carried out to confirm the
B. Parasitological examinations
C. Microscopic examination of stool
D. Biological test
E. * Coproculture
35. Patient K., 40 years old, hospitalized with the diagnosis of salmonellosis, with
symptoms of rise of body temperature to 40 °C, repeated vomiting, diarrhea.
Objectively: BP 80/40 mm Hg, pulse 130/minute, pale face, increased intestinal
A. * peristalsis, enlarged
Glucocorticoids, liver and spleen.
desintoxication What
therapy, is the best drugs
antibacterial treatment?
B. Diuretic, desintoxication therapy, antibacterial drugs
C. Enterorsorbents, diuretic, detsintoxication therapy
D. Glucocorticoids, diuretic, desintoxication therapy, antibacterial drugs
E. Glucocorticoids, diuretic, antibacterial drugs
36. Patient G., 22 years old, hospitalized in an infectious department complaining of
chills, stabbing pain in epigastrium, diarrhea, temperature increased up to
39,6 °C, repeated vomiting. Feces are copious, watery, without pathological
changes. 4 hours before the onset of symptoms ate the meat, salad. What is the
A. most appropriate
Antidiarrheal drugsdrug can be prescribed?
B. Quinolone group of antibiotics
C. Sulfanilamide group of antibiotics
D. Desintoxication therapy
E. * Enterosorbents
37. Patient K., 40 years old, hospitalized with the diagnosis of intestinal infection.
Complaining of general weakness, headache and diarrhea. Symptoms appeared
suddenly 2 hours after consumption of cake with custard. Suddenly there were
chills, nausea, profuse repeated vomiting, frequent diarrhea, abdominal pain and
short-term loss of consciousness, temperature increased to 38,6 °C. What is the
A. Salmonellosis, localized form, gastroenteritis type
B. Salmonellosis, localized form gastroenterocolitis type
C. Salmonellosis, localized form enterocolitis type
D. * Salmonellosis, generalized form
E. Salmonellosis, nosoparasitic

38. ?Patient C., 36 years old, periodically goes on a business trip in Egypt, is ill.
Complaints about the increasing body temperature up to 39,4 ?C with chills and
sweating, pain in the right hypochondriac region, emaciation. A light jaundice is
present. Increased liver, density and painful. In blood neutrophils leukocytosis,
A. increased ESR. USI revealed multiple liver abscesses. What kind of illness need
Legionellosis
B. Echinococcosis
C. Ascariasis
D. Liver cancer with metastases
E. * Amoebiasis
39. Student A., 22 years old, ill for 3 weeks, a month after returning from Ethiopia:
appeared abdominal pain, liquid excrement, abundant, up to 10 times per day,
in the form of “raspberry jelly”, was cramping in the abdomen, more right,
increasing in the bowel. Over time the disease has lost 6 kg body weight. Your
A. * diagnosis.amoebiasis
Intestinal
B. Shigellosis
C. Salmonellosis
D. Tumor bowel
E. Intestinal yersiniosis
40. The patient is 47 years old, desperately ill. Occasionally travels on a business
trip to Uzbekistan. Complaints about the increasing body temperature up to 39,4
°C with chills and sweating, chest pain and in the right hypochondriac region,
cough, bloody sputum. Skin with brown tint. Signs of rightside pneumonia. Liver
enlarged, painful. In blood neutrophils leukocytosis, increased ESR. X-ray of
chest shows infiltration of right lung tissue. USI of abdomen – multiple abscesses
A. Yersiniosis
B. Alveococosis
C. * Amoebiasis
D. Liver cancer with metastases in the lungs
E. Legionellosis
41. Patient C., 17 years old, working in vegetable shop. The disease began acutely,
2 days ago. This illness is connected with eating the dirty roots. The disease
started with chills, increasing the body temperature to 38,1 ?C, headache, pain
in muscles and joints of all groups, weakness, nausea. Cramping in the stomach
around the navel, in epigastrium, diarrhea up to 5 times. Excrements are liquid,
viscous, bed smell, normal colour. Objective inspection: scleritis, conjunctivitis,
A. * Yersiniosis
B. Dysentery
C. Salmonellosis
D. Cholera
E. Viral hepatitis
42. Disease started acutely with the complains of severe watery diarrhea, vomiting,
cramps in the lower extremities. Objectively: slow voice, sunken eyes, quick
pulse, low blood pressure, decrease of urination, weak cardiac sounds. In liver
A. and spleen are no changes. Put the correct diagnosis.
Salmonellosis
B. Dysentery
C. Food poisoning
D. * Cholera
E. Typhoid fever
43. Patient 18 years, hospitalized in an infectious department with diagnosis of
cholera, dehydration of IV degree. What measures are possible primarily?

A. Oral rehydration by glucose solutions


B. Tetracycline
C. * Intravenous stream introduction of salt solutions
D. Proceeding in the normal microflora of intestine
E. Intravenous stream introduction of sodium chloride solution
44. A Pakistani, 30 years old, severely ill: with the complains of frequent diarrhea
like rice water. Objectively: body temperature is 35,4 ?C, skin of peripheral parts
of the body cold on the touch, acrocyanosis, xerostomia, sharply decreased skin
elasticity and turgor. What should be done to assess the degree of dehydration.
A. Measurement of central venous pressure
B. Determination of blood urea and creatinine
C. Funduscopy
D. X-ray of abdomen
E. * Determination of specific density of blood plasma
45. Sick C., 20 years old, was admitted to the hospital with complains of the
frequent stool without stomach-ache, vomiting without nausea, pain in calf-
muscle. Objectively: Temperature of body is 36,2 °C. Skin is pale, cold, tongue is
dry, voice is horsed. The stomach is not painful at palpation. Stool looks like
A. * “rice-water”.
Triple What
negative are of
results the terms of discharging
bacteriological of patients
examination from the hospital at
of excrements
B. Double negative results of bacteriological examination of excrements
C. Single negative result of bacteriological examination of excrements
D. Single negative results of bacteriological examination of excrements and urine
E. Double negative results of bacteriological examination of excrements and urine
46. Patient B., 20 years, became ill acutely. The unexpected severe diarrhea
appeared with frequent vomiting, with plenty of vomit mass. The patient arrived
from a South-east Asia countries. He has temperature 36,1 °C during 3 weeks.
Abdomen is drawn-in, not painful. Stool looks like a rice-water. What the most
A. real changes
Increasing in blood
amount will be present?
of erytrocytes, leucocytes, relative closeness of plasma of
blood, hyperkalemia, metabolic acidosis
B. Decreasing of amount of erytrocytes, leucocytes, relative closeness of plasma of
blood, hypokalemia, metabolic acidosis
C. Decreasing of amount of erytrocytes, leucocytes, increase of relative closeness
of plasma of blood, hyperkalemia, metabolic acidosis
D. * Decreasing amounts of erytrocytes, leucocytes, relative closeness of plasma of
blood, metabolic hypokalemia acidosis
E. Increasing of amount of erytrocytes, leucocytes, relative closeness of plasma of
blood, hypokalemia, metabolic alkalosis
47. A sick, 29 years, repeated watery stool, frequent vomiting. Objectively: total
cyanosis, dryness of mucus membrane, turgor of skin is decreased. Temperature
of body is 35,2 °C. Pulse on radial artery is not determined. Tachypnea, muscle
A. spasm, urination
Dehydration of III is absent for 6 hours. What is the state of the patient?
degree
B. Dehydration of I degree
C. Infectious-toxic shock
D. Anaphylactic shock
E. * Uncompensated hypovolemic shock
48. For a patient, 35 y.o., the disease begun rapidly with a chill, increase of
temperature to 39 °C, vomitting, pain in epigastrium, diarrhea with the watery
stinking stool. 6 hours before the disease he ate a raw egg, potato with the
A. braised meat and drink juice. What exciter did cause the similar state probably?
Shigella
B. Collibacillus

C. Campylobacter
D. * Salmonella
E. Citrobacter
49. Patient C, 17 years old, worker of vegetable garden. Became sick 2 days ago.
The disease is related to using in food the carrot. Disease begun with chill, fever
of 38.1 °C, had pain in abdomen and in the muscles of all groups and joints,
weakness, nausea. Stool is liquid, viscid, stinking, ordinary color. At objective
examination: scleritis, conjunctivitis, hyperemia of soft palate, “raspberry”
A. Cholera
B. Dysentery
C. Salmonellosis
D. * Yersiniosis
E. Viral hepatitis
50. The disease began sharply, 6 hours ago at a normal temperature appeared
frequent liquid discharge, then vomit joined. At the examination: voice is
soundless, eyes are reddish, pulse - frequent, arterial pressure - low, urine is not
present, cramps appeared in lower extremities. The heart and lungs are without
changes. A liver and spleen are not enlarged. Choose the medications of
A. Tetracycline, erythromycin, chloramphenicol, gentamycine, ofloxacine
B. Erythromycine, chloramphenicol, gentamycine, ofloxacine, ciprofloxacin,
C. imodium
Tetracycline, erythromycine, chloramphenicol, benzyl-penicillin sodium salt,
D. * ofloxacine
Erythromycine, chloramphenicol, gentamycine, ofloxacine, ciprofloxacin
E. Erythromycine, gentamycine, ofloxacine, ciprofloxacine, imodium
51. The disease began sharply from diarrhea, that was accompanied by an
abdominal pain. The act of defecation brought facilitation. The stool is green
color, abundant, foamy, with a strong unpleasant smell. Temperature of body is
subfebrile. It is found out hyperemia and granularity of soft palate. In blood:
A. leucopenia, eozinophilia. Violations of water-electrolyte balance are moderate.
Cholera
B. Toxic food-borne infection
C. Salmonelliosis
D. Dysentery
E. * Rotaviral gastroenteritis
52. The seventh pandemic of cholera is caused by Vibrio cholera El Tor. It begans in
1961 on the Sulawesi island. However, to the epidemic of cholera arose up only
in the countries of the third world. It is known that cholera is classic bacterial
infection with the fecal-oral mechanism of transmission with the certain
infectious dose of exciter. After the L.V. Hromashevsky factors of transmission of
exciter of illness can be contaminated by them drinking-water, meal, hands.
A. Biological properties of exciter
B. Climate of country
C. * Social-economic conditions of population of country
D. Immune status of population
E. High development of industry and contamination of surrounding environment
53. Sick P., 55 years old, teacher, addmitted to the hospital with complaints for
attack-type of abdomen-aches, speeded up liquid defecating, T – 39 °C, false
urges on defecation, in the excrement are admixture of mucus and blood,
swelling of abdomen. Among the students of his class during the last week there
were cases of disease of shigellosis. What symptoms are most characteristic for
A. There is attack-type of abdomen-aches
B. * False urges on defecation

C. Admixtures of mucus and blood are in an excrements


D. Fever
E. Swelling of abdomen
54. Mother with a child hospitalized with the diagnosis of shigellosis, typical form,
moderate severity. Other family members are healthy. What disease measures
A. are conducted
Supervision in relation
7 days. to contact persons?
Non-permanent serum hemanalysis
B. * Supervision 7 days. Non-permanent bacteriological inspection of excrement on a
C. dysenteric group
Supervision 2 weeks. Non-permanent bacteriological inspection of excrement on
D. a dysentericduring
Supervision group 24 hours from the moment of exposure of patient
E. Hospitalization of contact persons on 7 days. Non-permanent serum
55. hemanalysis
Patient A., 23 years old, admitted to the infectious hospital through 6 hours after
the beginning of disease with complaints of a temperature rises to 40 °C,
nausea, repeated vomiting, pain in abdomen, frequent liquid excrement. At
examination a tongue is assessed a white raid. Sigmoid bowel is incrassate,
A. * painful. The diagnosis of shigellosis was set. What diseases can simulates the
Salmonellosis
B. Cholera
C. Acute pancreatitis
D. Acute adnexitis
E. Balantidiasis
56. 29 years old sick man complaints of a weakness, diarrhea. He rested on a south,
where cases of diarrhea were observed. Objectively: temperature of body is
36,4 °C, skin is clean, acrocyanosys, tongue is dry, an abdomen is soft, not
painful, defecating is abundant, watery, with the flakes of white color, odourless
A. * and admixtures. What is the preliminary diagnosis?
Cholera
B. Shigellosis
C. Giardiasis
D. Intestinal Yersiniosis
E. Salmonellosis
57. A patient is delivered with the previous diagnosis of shigellosis. State of patient
is of moderate severity, BP is 105/70 mm Hg, pulse – 90 per min. Pain is at
palpation in a right iliac region, sigmoid bowel is with spasm. Prescribe an
A. adequate treatment.
Chloramphenicol, sorbents, immunomodulators
B. Glucocorticoids, levomicetynum, rehydratation therapy
C. Chloramphenicol, diuretics, sorbents
D. * Furazolidonum, rehydratation therapy, sorbents
E. Furazolidonum, chloramphenicol, rehydratation therapy
58. Patient A., 65 years old, entered to the infectious department on the 3-d day of
disease with complaints for a rises of temperature to 39 °C, abdomen-ache,
swelling of abdomen, liquid stool with the admixtures of mucus and blood. He
does not know the reason of the disease. At examination – abdomen is not
A. tense; aofsigmoid
Cancer rectumbowel is incrassate, sensible. What is the preliminary
B. * Shigellosis
C. Amoebiasis
D. Salmonellosis
E. Yersiniosis
59. Man 28 years, fell ill suddenly: a chill, feeling of heat appeared, T – 38,5 °C,
attack-type pain is in the left iliac area, frequent liquid stool. Defecating has the
appearance of with-mucous mass. At palpation of abdomen a sickliness and
A. * spasm of sigmoid bowel is marked. What is the preliminary diagnosis?
Shigellosis

B. Amoebiasis
C. Ecsherichiosis
D. Heterospecific ulcerous colitis
E. Malignant tumor of colon
60. Patient S., 28 years old, became ill suddenly: a chill, feeling of heat, temperature
appeared to 38,5 °C, spastic pain in a left iliac area, diarrhoea. In defecation was
present blood-mucus masses. Objectively: pain in abdominal and spasm of
A. * sigmoid bowel. What is previous diagnosis?
Shigellosis
B. Amoebiasis
C. Ecsherichiosis
D. Nonspecific ulcerous colitis
E. Tumour of colon
61. At a patient who arrived from Crimea, at 5 o’clock in the morning began diarrhea
watery, without mucus and blood, later –only water. Vomit appeared in 12
hours. The temperature of body rose to 37,3 °C at the beginning of illness. An
A. abdomen-ache was not present. What is most credible diagnosis?
Ecsherichiosis
B. Salmonellosis
C. * Cholera
D. Food poisons
E. Shigellosis
62. Patient L., 32 years old, admitted in infectious department with complaints on a
chill, headache, expressed general weakness, acute pain in epigastrium,
diarrhea. Temperature – 39 °C, frequent vomits. Defecation is abundant, watery,
greenish color, without pathological admixtures. He ate meat, lettuce in a
factory dining-room before 4 hours to the disease. What drugs will be the most
A. Antibotulism serum
B. Salts and colloid solutions
C. Sulfanilamide preparations
D. * Quinolones
E. Sorbents
63. ?Patient 25 years, complaints of double vision, a decreasing of vision, shortness
of breath. Before the disease ate mushrooms of home preservation. Objective:
pallor, wide pupils with a weak reaction on light, dry mouth, a violation of
A. swallowing, flatulence, constipation. What is preliminary diagnosis.
Leptospirosis
B. Yersinioz
C. * Botulism
D. Giardiasis
E. Salmonellosis
64. In a hospital are admitted 5 patients in 15 hours. All of them have similar
complains of mild diarrhea and vomiting, diplopia, midriasis, visual disturbance,
difficulty in swallowing, dry mouth, difficulty in breathing. All patients together
joined in a dinner party, used different dishes, including meat, salad, canned
A. mushrooms. The most likely diagnosis.
Cholera
B. * Botulism
C. Typhoid fever
D. Salmonellosis
E. Rotaviral gastroenteritis

65. Patient with complaints about a general weakness, diarrhoea appeared to the
doctor. Objectively: temperature of body 36,4 °C, skin covers clean,
acrocyanosis, tongue is dry, stomach is soft, not painful, emptying is abundant,
watery, with the supernatant flakes of white color, odourless and admixtures. For
A. clarification
From blue toof diagnosis culture of stool was made on Resselya. What is the
green
B. From yellow to green
C. From green to yellow
D. From yellow to blue
E. * From blue to yellow
66. Citizen of Pakistan, 30 years became ill rapidly with frequent watery stool which
appears like a rice water. Objectively: temperature of body 35.4 °C, skin is cold,
acrocynosis, elasticity of skin and turgor is decreased. By what method may
A. estimate
To examine thean
degree of dehydration?
eyeballs
B. To check central venous pressure
C. To define the level of urea and creatinine in blood
D. * To check the loss of blood plasma
E. To check the pressure of blood
67. For a patient E., 37 years old, a disease began rapidly, 6 hours the frequent
liquid emptying appeared onto the plain rise of normal temperature of body,
vomit joined then. At an inspection: aphonia, eyes are hollow, pulse frequent,
threadlike, low blood pressure cramps appeared in lower extremities. Liver and
A. spleen are not
Tetracyclin, enlarged. Choose
erythromycin, preparations
chloramphenicol, for starting or
ciprofloxacin etiotropic
imodiumtherapy.
B. Erythromycin, chloramphenicol, benzylpenicillin or imodium
C. Tetracyclin, erythromycin, chloramphenicol, benzylpenicillin or ofloxacin
D. Tetracyclin, erythromycin, chloramphenicol, or bifi-form
E. * Chloramphenicol, erythromycin or ciprofloxacin
68. For a patient E., 37 years, bodyweight of 70 kg, frequent liquid emptying
appeared with rise of body temperature, frequent vomiting joined in later. At an
inspection: aphonia, eyes are hollow, pulse with frequent threadlike, blood
pressure is not determined, tachypnea, total cramps. Liver and spleen are not
A. * enlarged.
7 L What volume of salt solutions must be entered for primary
B. 3,5 L
C. 5L
D. 10 L
E. 2L
69. Sick C., 23 years, ill from 3 days after returning from India. The disease has
begun with the liquid emptying which looks like a watery, after wards joined with
multiple vomiting, expressed weakness and cramps. The state progressively got
worse and was admitted within 12 hour in an infectious hospital in a grave
condition. The eyes of person is sharped, skin is cold, acrocynosis temperature
of body 35,5 °C. Aphonia, cramps of hands and feet. The fold of skin falls out
through abdomen. stomach is pulled in, no pain at palpation. Blood pressure is
A. Intestinal echerihiosis
B. Salmonellosis
C. Shigellosis
D. * Cholera
E. Amebiasis, intestinal form

70. Sick I., 25 years, returned from rest in Egypt. Early in the morning diarrhea
began. Emptying each 1-1,5 hrs, abundant watery, without mucus and blood.
Abundant vomiting also appeared. The temperature of body at first rise to 37,3
°C, afterwards became 35,5 °C, stomach-aches negative. Delivered in an
A. infectious department.
Treatment with sorbentsWhat is first aid?
B. Polyhybrid solution intravenous
C. 5 % solution of glucose intravenous|
D. Fresh-frozen plasma intravenous
E. * Salt solutions intravenous
71. Sick, 20 year old, apeared to the hospital with complaints about the frequent
emptying without a stomach-ache, vomit without nausea, pain in calve.
Objectively: temperature of body – 36,2 °C. A skin is pale and cold, a tongue is
dry, voice getting hoarse. A stomach is pulled in, painless, emptying like a “rice-
A. water”. negative
Double What arebacteriological
the criterias forexamination
dischargingof
ofexcrements
the patient from the hospital?
B. Single negative bacteriological examination of excrements
C. * Triple negative bacteriological examination of excrements
D. Negative bacteriological examination of excrement and urine
E. Double negative bacteriological examination of excrement and urine
72. Patient E., 47 years, became ill in 4 days after returning from Egypt. A disease
begun with the liquid stool which looks like a watery, joined with frequent
vomiting, expressed weakness and fatigue. The state progressively got worse
and within 18 hrs was admitted in an infectious department in a very severe
condition. The skin is cold, cyanosis, temperature of body is 35.5 °C. Aphonia,
cramps of hands and legs. The turgor of skin is decreased. A stomach is drawn-
in; no pain at palpation. Blood pressure is 70 and 30 mm of Hg. The pulse on
radial artery is not determined, urination and stool are normal. Weight of patient
A. I
B. II
C. * IV
D. III
E. V
73. Sick X., which returned from vacations from Turkey within 5 hrs in the morning,
diarrhea began. Stool every 1-1.5 hrs, with abundant watery feces without
mucus and blood. In future bloodless watery, abundant vomiting appeared
through 12 hrs. The temperature of body at first rise to 37.5 °C afterwards
A. became 35.7 °C is admitted in an infectious department. What disease did you
Echeriosis
B. Food poisoning|
C. * Cholera
D. Salmonellosis
E. Balantidiasis
74. A farmer O., 50 years old, hospitalized in a moderate condition with complaints
about dryness in mouth, multiple vomitings, pain in the epigastriums and
A. frequent watery oxygen
Hypersaturated stool. First aid to the patient is?
B. Transfusion of fresh-frozen plasma
C. Tetracyclin
D. * Intravenous introduction of salt solutions
E. Introduction of polyhybrid

75. Sick P., 25 years old, presents with frequent vomitting. Objectively: dryness of
skin and mucous membranes, brief cramps in gastrocnemius muscles, the
temperature of body is normal, voice is hoarsed, moderate tachycardia and
hypotension. The compensated metabolic acidosis is marked. About what
A. degree of dehydration is it possible to think?
IV
B. II
C. * III
D. I
E. There is no dehydration
76. An unconscious patient is delivered in the intensive department. Pale dark
circles around eyes. Skin is cold with sticky sweat. Temperature of body 35,6 °C.
Pulse 140/min and weak. Blood pressure is 40 /0 mm of Hg. Tongue is dry.
Emptying is involuntary and “watery”, vomited twice. What infusion must be
A. given as intensive therapy.
Albumin
B. Rheopoliglykin
C. 5 % glucose solution
D. * Polyionic salt solutions
E. 10 % glucose solution
77. Patient P, 35 years old, became ill within 2 days after returning from India. A
disease begun with appearance of liquid diarrhea which resembled a watery
appearance, afterwards joined with multiple vomit, expressed weakness and
fatigue. State progressively got worse and within 12 hrs delivered in an
infectious department in a grave condition. The skin is cold, cyanosis,
temperature of body 35.5 °C. Aphonia, cramps of hands and feet. A skin fold
falls out from abdomen. The stomach is pulled in, no pain at palpation. Blood
pressure is 70 and 30 mm of Hg. A pulse on a radial artery is not determined,
A. 10 liters
B. * 8 liters
C. 4 liters
D. 6 liters
E. 2 liters
78. A patient L., who returned from Crimea, developed diarrhea at 5AM. Bowel
movements are each 1-1.5 hrs, watery, without mucus and blood. In 12 hrs a
single episode of vomiting developed. The temperature of body at first rise to
37.3 C, stomach-aches is present. he was examined by the doctor of first-aid
and delivered to an infectious isolation with the diagnosis of acute intestinal
A. Intestinal echerihiosis
B. Salmonellosis
C. Echeriosis
D. Food poisoning
E. * Cholera
79. The ill patient in severe state was delivered to infectious department with no
consciousness. Pale dark circles around eyes. Skin is cold, covered with sticky
sweat. Temperature of body 35.6 °C. Pulse 140/min and weak.Blood pressure is
40 and 0 m of Hg. Tongue is dry. Emptying is involuntary and watery. Three
A. times vomited like “fountain”.What is the state of the patient?
Collapse
B. Infectious toxic shock
C. * Dehydration shock
D. Cereblral edema
E. Intoxicated encephalopathy

80. A patient with severe form of botulism received specific therapy with
antibotulinic serum. There is no sensibilization after intra- and subcutaneous
tests. During introduction of basic dose of preparation the patient suddenly stop
of breath, tachycardia and cyanosis appeared. The patient was transported to
A. * intensiveof
Progress care department.
basic disease Ps – 110/min., ÀP – 140/60mmHg. The reason of this
B. Anaphylactic shock due to introduction of antibotulinic serum
C. Anaphylactic shock due to introduction of infusive media in which antibotulinic
D. serum is present of pulmonary artery branches
Thromboembolia
E. Acute cardiac insufficiency
81. Patient D., 13 y.o., hospitalized with complaints of nausea, abdominal pain, liquid
emptying without pathological admixtures with an unpleasant smell, 6 times per
day. The day before he had a supper with meat salad. What is a first aid?
A. Peroral rehydration by glucose-electrolytes solutions
B. * To wash a stomach and intestine by solution of hydrocarbonate of sodium
C. Antibacterial preparations of wide spectrum of action
D. Renewal of normal microflora of intestine
E. To wash a stomach and intestine by solution of permanganate of potassium
82. Patient H., 37 y.o., appealed to the doctor on the second day of disease with
complaints of the promoted fatigueability, weakness in muscles („cotton feet”),
violation of sight, dryness of mouth. Works in a tourist agency, often is in the
oversea business trip. Three days ago was with friends on a picnic in a forest,
where ate the varied meal of the domestic making. At a review: Ò-36,8 °C.
Blepharoptosis. Midriasis. The reaction on light is lost. A soft palate is not
A. The sick man
B. Man-bacteria carrier
C. * Domestic animals
D. Canned foods
E. Ticks
83. The patient, 45 y.o., entered clinic on the 2nd day of illness with complaints of a
weakness, diplopia, dryness in mouth, constipations. 3 days ago ate the smoked
pork of the domestic making. At a review: skin pale, consciousness is clear.
Temperature 37,2 Ñ, pulse 68 in 1 min, AP 120/80, pupils are widening, reaction
on light slow, ptosis, horizontal nystagmus. Paresis of soft palate. A
A. Sulfanilamides
B. Antibiotics
C. Salts solutions
D. Antiviral preparations
E. * Antibolutilic antitoxic serum
84. A geologist 36 y.o. is hospitalized with complaints of a general weakness,
dryness in mouth, diplopia and net before eyes. The day before used canned
meat of the domestic making. Objectively: bilateral ptosis, expansion of pupils,
paralysis of accommodation, violation of convergence. A mucus membranes are
dry, swallowing is difficult, voice hoarse, disarthria. An abdomen is soft, some
A. Hyperergic inflammation of intestine of Artius type
B. Acute adrenal gland insufficiency
C. Acute violation of cerebral blood circulation
D. * Paralytic action of exotoxin on parasympathetic innervations
E. Degenerative changes of myelin shells of nervous fibres

85. Family (three persons) came to permanent establishment with the disease which
is connected with the use of fish can food of the domestic making. These
patients had a similar clinic: violation of sight, clouds before eyes, impossibility
to read a small font, they appealed to the oculist. What link of pathogenesis is
A. * characteristic
Blocking for thisof
of selection disease?
acetylcholine from synapses
B. Blocking of inserted motoneurons
C. Demyelinization of nervous fibres
D. Stimulation of synthesis of adenilacyclase
E. Development of hypocoagulation
86. Patient M, 32 y.o., entered clinic on 3rd day of disease with complaints of
nausea, feeling of weight in abdomen, vomits, liquid stool. Then appeared
clouds before eyes, doubling of objects, voice hoarse, violation of swallowing. A
day before he used the dried fish. During review: state is moderate., violation of
active motions like paralyses. There is also midriasis, vertical nystagmus,
blepharoptosis, absent reaction of pupils on light. Tongue is dry. Flatulence of 2
A. Reaction indirect hemaglutination
B. Immunofluorescent
C. Virology
D. Reaction of coaglutination
E. * Reaction of neutralization (biological test)
87. Patient, 32 y.o., complains of progressing muscular weakness, worsening of
sight, doubling of objects, “net”, before eyes, violation of swallowing (can not
swallow a hard meal), thirst. In the first day of illness single liquid stool was
present without pathological admixtures, nausea. 2 days prior to beginning of
disease was in guests, used an alcohol, canned mushrooms. Presence of ptosis,
A. Mixture of serums of types A and Å for 5 thousands IU and type B 10 thousands
B. of IU
Mixture of serums of types A, B and Å for 10 thousands of IU
C. Mixture of serums of types A, B and Å for 5 thousands of IU
D. * Mixture of serums of types A and Å for 10 thousands IU and type B 5 thousands
E. of IU
Mixture of serums of types A and B for 10 thousands IU and type Å 5 of thousand
88. of IU
Patient, 40 y.o. in 5 hours after the use in the meal of canned mushrooms of the
domestic making a sharp general weakness, nausea, vomits, dryness of mucus
membranes of oral cavity, doublings of objects, disorders of act of swallowing.
A. Diagnosis?by Belladonna
Poisoning
B. Diphtherial polyneuritis
C. * Botulism
D. Rotaviral infection
E. Poliomyelitis
89. Woman L, 65 y.o., became ill sharply, in 12 hours after the use in the meal of
canned mushrooms of the domestic making and fried eggs fried on fat. A sharp
weakness, nausea, double vomits, appeared „clouds” before eyes, disorders of
swallowing. At a review: Ò-36,2 C., ptosis, midriasis,anizocoria, inspiratory
A. * dyspnea. What disease is present in women?
Botulism
B. Salmonelosis
C. Poisoning by mushrooms
D. Sharpening of chronic cholecystitis
E. Toxic food-borne infection

90. Sick E., 21 years old, grumble about nausea, insignificant dull or rarely
aggressive pain in a stomach. Appetite is decreased. Periodically presence of
flatulence, grumbling in a stomach and semi-fluid emptying. Sick during about 6
month. The diagnosis of chronic cholecystitis was proposed. In examination
pallor of skin, lowering of mass of body and signs of vegetative dystonia are
A. * Giardiasis
B. Salmonellosis
C. Amebiasis
D. Balantidiasis
E. Intestinal ºcheriosis
91. Diagnosis of giardiasis is based on the discovering of cyst in fresh incandescence
and vegetative forms in duodenal content. At diarhoea in incandescence can
appear trophozoite on faecal microscopic examination and preparations with
solution of Lugola and åîsin. Examination is more expedient to conduct 3-5 times
A. * at intervals
By 1-2ofdays.
application How to increase frequency
the formalin-åpiniphrine of findings?
enriching methods
B. Cultivation in thermostat
C. Cultivation in an anaerobic chamber
D. By the method of floatation in bilious clear soup
E. To sow on a nourishing environment
92. Patient M., 32 years old, during a month has disturbing cough, increasing of
temperature up to 38.0 °C. In anamnesis ascaridosis was found. On x-ray was
found separate homogeneous infiltration without clear contour of bronchus. In
A. * blood test –infiltration
Eosinophil åîsinophil(Lefler
is 55 %. What most probable diagnosis?
syndrome)
B. Plague
C. Chronic bronchitis, acute phase
D. Acquired pneumonia
E. Tumour of lungs
93. Veterinarian 57 years old, on the 3rd day of disease appealed to the doctor with
complaints of headache, high temperature, pain in gastrocnemius muscles,
icterus, dark urine and diminishing of its amount. Objectively: temperature of
body is 38 °C, sclerotic big peteachia on overhead part of thorax,
A. * hepatospleenomegaly. Most reliable preliminary diagnosis will be:
Leptospirosis
B. Brucellosis
C. Viral hepatitis
D. Pseudotuberculosis
E. Trichinosis
94. A sick P., 40 in 2 weeks after eating of uncooked pork, purchased at the
elemental market, has sudden rise of temperature upto 40 °C, myalgias
appeared, oedematous, papular rash on extremities and trunk and dry cough.
Accepted aspirin. Blood test, leucocyte – 12?109 eosinophil– 40 %. What disease
A. * it is needed to think about?
Trichinosis
B. Ascaridosis
C. Leptospirosis
D. Teniosis
E. Allergic reaction

95. Woman 24, complains on heaviness in the area of liver during 3 days, with an
irradiation in the back. Signs of icterus are not present. Last menstruation – 5
weeks ago. Appointed ultrasonic research of abdominal region and small pelvis
was, found marked progress uterogestation. Expansion of common biliary tract,
inflammation of gall-bladder, bulge of its its wall, presence of crystal sediment,
extended loops of small intestines are visualized. Investigation of colonoscopy
found additional exogenic structures as a “ribbon”, which is displaced in
A. * Ascaridosis
B. Partial intestinal uncommunicating
C. Sharp calculary cholecystitis
D. Choledocholithiasis
E. Cholecystitis
96. Sick, 37 years, acted on stationary treatment with complaints about bad feel,
headache, increase of temperature upto to 39.5 °Ñ, edema, pains in muscles of
eyes, tongues, lower extremities. 2 weeks prior to beginning of disease used raw
pork fat. Objectively: edema present. The symptom of parstersky’s is negative
from both sides. In a blood test: Leucocytes – 18.5*109 L, eosynophils – 22 %,
basophils – 2 %, neutrophils – 56 %, stab neutrophills – 16 %, monocytes – 4 %.
A. * Trichinosis
B. Leptospirosis
C. Acute nephritis
D. Quike’s edema
E. Dermatomyositis
97. Patient 35 years old, was entered to the clinic with complaints about grumbling
in a stomach, propensity to diarrhea, that are already present during 2 months.
After the last two weeks at the act of defecation noticed moving snatches of flat
struture. Has habit to use in a meal the raw stuffing from meat of home animal
with spices. At colonoscopy there are numerous lateral folds which fills almost all
A. Mebendazol
B. Combantrin
C. Albendazol
D. * Phenasalum
E. Medamin
98. Patient, 27 years old, appeared with complaints about itching in perianal area,
which is present in evening before sleep. It continues for 1-3 days and
disappears independently, but reconvolense afterwards.
A. t probable diagnose will be:
B. * Trichinosis
C. Enterobiosis
D. Ascaridosis
E. Helminthosis
F. Cystitis
99. Patient A, 29 years old, appealed with complaints about the decline of appetite,
nausea, vomit, stomach-ache, diarrhea pain of chest, weakness, dizziness,
parahypnosis.There is present hypochromic anaemia.Periodically used raw and
A. half cooked raw pork. What most probable diagnosis?
Teniarinhosis
B. Difilobotriosis
C. * Teniosis
D. Trichinellosis
E. Ascaridosis

100. At a child with the clinical displays of ARVI a generilized lymphadenopathy, one-
side conjunctivitis, increasing of liver and spleen are marked. Most reliable
A. diagnosis?mononucleosis
Infectious
B. Leptospirosis
C. * Adenoviral infection
D. Flu
E. Pseudotuberculosis
101. A patient 14 years old, hospitalized in the infectious department in severe
condition with considerable headache mainly in frontal and temporal area, pain
in eyeballs, in muscles and joints. Objectively: patient is excited, temperature of
the body is 39 °C. Bradycardia changed by tachycardia. Muscles tonic and clonic
cramps. Positive meningeal signs. It is found in epidemic anamnesis, his brother
A. * Flu with pneumonia and edema of brain
B. Flu, typical course
C. Parainfluenza, false croupe
D. Respiratory-sencytial infection
E. Adenoviral infection, pneumonia
102. A patient H., 22 years old, with flu was hospitalized into infectious department
with the acute worsening of the common state. Consiousness is stored. The
patient strangles. Pallor of skin with cynosis. Respiratory rate 50 per min, AP
80/55 mmHg, pulse 110 per a min, temperature 39.8 °C. During percussion of
lungs tympanic sound with dullness in lower quadrant was found. Crackles in the
A. Bronchitis
B. * Edema of lungs
C. Edema of brain
D. Infectious-toxic shock
E. Meningoencephalitis
103. A sick, 54 years old, hospitalized in infectious department in the grave condition.
Complaints about expressed headache, mainly in frontal and temporal areas
superciliary arcs, origin of vomiting on peak of pain. Objectively: patient is
excited, temperature of body 39 °C, AP 100/60 mm Hg. Bradycardia changed to
tachycardia. Tonic cramps, meningeal signs appeared. From anamnesis it is
A. * Mannitol, lasix, prednisolone, åuphyllin, suprastin
B. Mannitol, acetophene
C. Lasix, analgin, ampicillin
D. Verospiron, euphyllin, demidrol
E. Aspirin, analgin, demidrol
104. A patient becomes sick very fast: chills, increasing of temperature to 40,1 °C,
headache in frontal and temporal regions, pain in eyeballs, close nose, dry
cough and pain in the chest. The nose bleeding, nausea, double vomits.
Objectively: conjunctivitis, hyperemia, edema, hemorrhages in mucous of
otopharhynx, tachycardia. Blood pressure is low. Difficult breathing. What is the
A. Meningococcemia
B. Epidemic typhus
C. Leptospirosis
D. * Flu
E. Typhoid fever
105. A patient 20 years old fell ill rapidly with increasing of temperature to 39.9 °C.
complains about headache in frontotemporal region, pain in eyeballs, dull ache
in whole trunk, closed nose, scrapes in the throat, dry cough. There was nose-
bleeding. What diagnosis is most possible?

A. * Influenza
B. Adenoviral infection
C. Parainfluenza
D. RS-infection
E. Enteroviral infection
106. A sick person, 45 years old, was hospitalized after 2 days of disease. On Sunday
he came back from India (sailor). Complains about increasing of temperature to
41 °C, severe headache, shortness of breath, cough, with sputum. Objectively:
pallor, cyanosis of mucous, tachycardia. Breathing is weaken, crackles in the
A. * Flu
lower-back partsby
complicated of pneumonia
the lungs, crepitation. What is the possible diagnosis?
B. Miliary tuberculosis
C. Plague, pulmonary form
D. Leptospirosis
E. Sepsis
107. A 25 years old patient, fell ill rapidly, with chills and temperature rose to 39,9 °C,
headache appeared in frontotemporal regions, pain in eyeballs, dull pain in all
trunk, closed nose. Dry cough after 2 days of illness, there was nose bleeding.
Objectively: hyperemia with sputum, isolated petechial rash and shallow
A. grittiness of soft palate. Difficult breathing in lungs. What is the most possible
Leptospirosis
B. Adenoviral infection
C. Typhoid fever
D. * Flu
E. Epidemic typhus
108. A patient 20 years old, complains about increasing of temperature up to 39 °C,
headache in frontal area, pain in eyeballs, photophobia, pain in muscles, dry
cough. Became ill sharply. Objectively: severe state. Face is hyperemic, eyes
brilliant, injections of scleras. Pulse 96/min, rhythmic, tones of heart are
hyposthenic. Meningeal symptoms are not present. Blood analysis: leuk 9x10*9,
A. * Ifluenza
B. Adenovirus infection
C. Leptospirosis
D. Pneumonia
E. Epidemic typhus
109. A patient 26 years old, became sick rapidly: temperature 39.5 °C, severe
headache, mainly in frontal and temporal areas, pain in muscles and joints.
Examined on the 2nd day of illness: state of middle weight, skin is clean.
Moderate hyperemia with cyanosis, pulse 120 per min, rhythmic. Heart activity
A. is rhythmic, tones are muffled, in lungs there is vesicular breathing. What is the
Aspirin
B. * Rimantadin
C. Ampicillin
D. Ascorbic acid
E. Ribonuclease
110. A patient fell ill very rapidly: chills, increase of temperature to 40.1 °C, headache
in frontotemporal regions, pain in eyeballs, close nose, dry cough, and chest
pain. Nose bleeding, nausea, vomiting appeared after 4 hours. Objectively:
conjunctivitis, hyperemia, edema, point hemorrhages in mucus of epiglottis,
tachycardia. Blood pressure is low. Weaken breathing in the lungs. What is the
A. Leptospirosis
B. Epidemic typhus
C. * Flu

D. Ìeningococcemia
E. Enteroviral infection
111. A student, 18 years old, for 7 days complains about weakness, hyperthermia to
37.8 °C, mucous excretions from a nose, pharyngalgia at swallowing, pain in
eyeballs. Objectively: increased lymph nodes of neck and mandible,
lymphadenitis, edema and injection of conjunctiva, hyperemia of mucous of
A. * Adenoviral
epiglottis, hypertrophy
infection of tonsils. What is the most reliable diagnosis:
B. Influenza
C. Infectious mononuleosis
D. Rhinoviral infection
E. Parainfluenza
112. Patient B., 20 years old, complains about severe headache in temples and orbits,
dull ache in the trunk, dry cough. Temperature of the body 39.6 °C.
Inflammatory changes of mucous membrane of oropharynx. Normal breathing in
A. the lungs. What is the most credible diagnosis?
Pneumonia
B. Parainfluenza
C. Respiratory micoplasma
D. * Flu
E. Meningococcal infection
113. Sick, 52 years old, with complaints about pain in lumbar region, headache
edema of chin. It is known from anamnesis that the sick suffers from obesity of
²² degree. Recently carried heavy neurological stress and had flu. He has chronic
bronchitis for 5 years, chronic gastritis for 8 years. Objectively: Temperature of
the body 38.2 °C, AP – 140/90 mm Hg. It is proposed the diagnosis of acute
A. Neuro psychologic stress
B. Chronic bronchitis
C. Chronic gastritis
D. Obesity
E. * Flu
114. A patient C., was hospitalized on the 2nd day of illness with complaints about
hoarseness of voice, rough barking cough, labored breathing. Objectively: the
state is severe, uneasy, pallor, temperature 37.1 °C, BR 30/min., breathing is
noisy, can hear from the distance, with participation of auxiliary musculature.
A. Which virus
Rhino viruses could cause development of similar state?
B. Influenza virus
C. Adenovirus
D. * Parainfluenza virus
E. Cytomegalovirus
115. A sick woman, 42 years old, complaints about temperature 39.3 °C, headache in
the frontal area, pain in the eyeballs, photophobia, pain in muscles, dry cough.
Became ill suddenly one day before. Objectively: state is severe. Hyperemia of
the face, eyes shinny, injection of scleras. Pulse 96/min., rhythmic. Tones of
heart are hypotonic. Both lungs are dissipated. Dry wheezes. Mucosa of
epiglottis is hyperemic, grainy, vessels are extended. Meningeal symptoms are
A. * Flu
B. Measles
C. Meningococcal infection
D. Pneumonia
E. Epidemic typhus

116. A patient, 75 years old, called a doctor to home. Rashes and subfebrile
temperature, general weakness, pharyngalgia, conjunctivitis. In family a child is
ill with acute adenoviral disease. A patient considers himself ill on the second
day. At a review there are signs of pharyngitis. There are enlarged lymphatic
nodes: of neck, front and back, armpits and inguinal up to 1 cm in diameter,
soft. Pharynx is hyperemic, tonsils are hypertrophy and hyperemic. Both lungs
have wheezing sounds. Not clean breathing. Tones of heart are muffled. AP
A. * Acute adenoviral infection
B. Flu
C. Hepatitis B
D. Infectious mononucleosis
E. Hepatitis A
117. A child of age 2 years has temperature of body 37.3 °C, cold, hoarse voice
“barking cough” appeared suddenly the anxiety, shortness of breath, appeared
with participation of auxiliary muscles. Supposed diagnosis?
A. * Parainfluenza, false croup
B. Diphtheria croup
C. Allergic laryngitis, croup
D. Flu, laryngitis
E. Acute exudative pleuritis
118. A patient of 5 years old, which treated at home on an occasion of flu by aspirin,
calcium gluconatis, on the second day from the beginning of disease “coffee
grounds” vomiting appeared, melena. What complication arises?
A. Neurotoxicosis
B. Pneumonia
C. * Hemorrhagic syndrome
D. Infectious-toxic
E. Bowel obstruction
119. A patient with flu complicated by pneumonia, during some days there are the
displays of infectious-toxic shock of ²² degree. In biochemical blood analysis the
level of urea and creatinine increases. What from these preparations is not
recommended to enter in such a situation?
A. * Adrenalin
B. Prednisolone
C. Polioniic solutions
D. Dofaminum
E. Heparin
120. A 23 years old person, became ill sharply: fever 38.2 °C, moderate diffuse
pharyngalgia at swallowing, pain and itching in the right eye. Objectively:
tonsillitis, pharyngitis, conjunctivitis. What is previous diagnosis?
A. * Adenoviral infection
B. Enteroviral infection
C. Parainfluenza
D. Flu
E. Acute respiratory infection

121. A patient 17 years, 11th class student, were a lot of cases of ARI (acute
respiratory infection) have happened, appealed to a doctor in clinic at 3rd day of
disease with complaints of chills, general weakness, a moderate sore throat,
running nose, swelling of face, watering from eyes Objective examination: minor
palatal hyperemia brackets and tonsillitis, on a background of moderate edema
of tissues. Conjunctivitis. During palpation not painful enlarged inframaxillary
lymph nodes, and enlarged neck lymph nodes were found. Crepitation can’t be
A. Diphtheria
B. * Adenoviral infection
C. Meningococcal nasopharyngitis
D. Influenza
E. Infectious mononucleosis
122. Patient R., 16 years old, hospitalized for 5-day illness with complaints of
moderate headache in fronto-temporal region, laid nose, sore throat, pain in the
left eye, rise in temperature to 38.1-38.5 °C. General condition is satisfactory.
Shortness of nasal breath, mucous discharging from the nose, hyperemia of
face, enlargment of the neck and submaxillary lymph glands, left foamy
A. Influenza
B. Infectious mononucleosis
C. Enteroviral infection
D. * Adenoviral infection
E. Influenza
123. Patient R., 26 years old, became ill sharply: temperature 39,5 °C, severe
headache, mainly in frontal and temporal an area, pains in muscles and joints.
Examined on the 2th days of illness: state of middle weight, skin clean, dry.
Moderate hyperemia with cyanosis, pulse 120/min., rhythmic. Cardiac activity
rhythmic, tones are muffled, in lights of the vesicular breathing. Stomach is
A. Aspirin
B. * Remalol
C. Ampicillin||
D. Ascorbic acid
E. Ribonuclease
124. A patient A., 13 years old, on the 4th day of illness a district doctor marked such
subjective and objective data: insignificant indisposition, mild headache,
hoarseness of voice, itching in throat, barking dry cough, temperature of the
body 37,4 °C. Pulse 86/min., difficult nasal breathing, insignificant serous
A. excretions from nose. Which acute respiratory infection does the patient carry?
Influenza
B. RS-viral infection
C. * Parainfluenza
D. Adenoviral infection
E. Enteroviral infection
125. A child 3 years old is found in the grave condition – forced breathing, dry barking
coughing, voice is hoarse, perioral cyanosis. The third day, temperature of body
is subfebrile, mild common cold. In lungs single dry wheezes can be heard.
Moderate tachycardia. For which disease these symptoms are characteristic?
A. Localized diphtheria of oropharynx
B. Whooping-cough
C. * Parainfluenza, false croup
D. Bronchopneumonia
E. Adenoviral infection

126. To the district doctor a patient, complaints on abundant excretions from a nose,
moderate headache, hearing loss, perversion of taste. On examination – dry of
skin, nose excoriation, in a pharynx – mild hyperemia. Temperature of body is
subfebrile. Pathological changes of internal organs are absent. Which acute
A. respiratory infection
Adenoviral viral infection carries the patient?
B. Parainfluenza
C. * Rhinoviral infection
D. RS-infection
E. Influenza
127. In a child with the clinical display of acute respiratory viral infection observed
generalized lymphadenopathy, one-sided conjunctivitis, increase of liver and
spleen. What will be the most credible diagnosis?
A. Infectious mononucleosis
B. Leptospirosis
C. * Adenoviral infection
D. Influenza
E. Meningococcal infection
128. A patient admitted in the infectious department with diagnosis of acute
respiratory viral infection. Became ill suddenly, the disease is accompanied by
the increase of temperature of body till 39 °C, by severe headache, mainly in
area of frontal, temporal, above eyes, dryness in nose, itching in throat, dry
cough, dull pain in all body. He had bleeding from nose twice at home. Which
A. Adenoviral infection
B. RS-infection
C. * Flu
D. Parainfluenza
E. Enteroviral infection
129. Patient L., 18 years old is sick with fever till 38 °C which proceeds 5 days. he has
moderate dry cough, common cold, badly opens eyes. On examination –edema
on face, expressed conjunctivitis with film raids. Mucous of pharynx is
hyperemic, posterior wall of pharynx is grainy. Internal organs are without
A. pathology.
Viral What form of disease does the described picture correspond to?
conjunctivitis
B. Allergic dermatitis
C. * Adenoviral infection
D. Influenza
E. Rhinoviral infection
130. In a patient of 16 years old, the disease began gradually, from the catarrhal
syndrome. For 2-3 days the temperature of body increase till 38,5 °C, cold,
severe cough with the negligible quantity of mucous sputum, «souring» eyes.
Peripheral lymph nodes are soft, painless and some enlarged. Mucous of
pharynx is hyperemic, granules on posterior part of pharynx. General state is
A. Ascorbic acid
B. Aspirin
C. * Desoxyribonucleas
D. Remantadin
E. Aminocapronic acid

131. Patient 22 years old, has increase temperature of body till 37,8 °C. Treated
under the supervision of district doctor with a diagnosis of influenza. On the 5th
day of illness temperature remained the same; it began difficultly in opening
eyes. On examination – edema on face, expressed conjunctivitis with film
stratifications. Mucous pharynx is heperemia, on the back wall of gullet
considerable graininess. Lymph nodes are enlarged in neck. The general state of
A. Leptospirosis
B. Infectious mononucleosis
C. * Adenoviral infection
D. Allergic dermatitis
E. Meningococcal infection
132. A patient P., 14 years old, is hospitalized in the infectious dept. in grave
condition. Complaints on headache, mainly in frontal and temporal regions,
superciliary arcs, vomiting on severe pain, pain on movement of eyeballs, in
muscles, joints. Objectively – a patient is excited, temperature of the body 39
°C. BP 100/60 mmHg. Bradycardia was replaced by tachycardia. Tonic cramps
appeared. Doubtful meningial signs. From anamnesis it is found that at home his
A. Verospiron, euphyllin, dimedrol
B. * Mannitol, paracetamol, prednisolone, euphyllin
C. Analgin, dimedrol, aspirin, ampicillin
D. Mannitol, aspirin
E. Lasix, analgin, ampicillin
133. Patient P., 14 years old, is hospitalized in the infectious department in the severe
condition. Complains on expressed headache, mainly in frontal and temporal
regions, supercilliary arcs, origin of vomiting appear in condition of severe pain,
pains by moving the eyeballs, in muscles and joints. Objectively: patient is
excited, body temperature-39 °C. BP-100/60 mm Hg. Bradycardia was replaced
by tachycardia. Appeared tonic cramps. Doubtful meningeal signs. From
A. Influenza, typical flow
B. * Influenza with the phenomena of edema of brain
C. Respiratory-syncytial infection
D. Parainfluenza
E. Adenoviral infection
134. Patient M., 11 years old, complains on general weakness, cough, at night
suddenly temperature rose till 39,5 °C, appeared restlessness, barking cough,
noisy whistling breathing with drowing in supra- and subclavicular cavities,
intercostal spaces. He was in contact with the patient acute respiratory viral
A. * Prednisolon,
infection. What
hotshould recommend him the first line?
foot-baths
B. Seduxsen, euphylin
C. Euphylin, vitamin C
D. Antibiotics, dimedrol
E. Astmopen, diazolin
135. A sick, 29 years, emptying watery stool repeatedly, frequent vomiting.
Objectively: total cyanosis, dryness of mucus membrane, turgor of skin is
decreased Temperature of body 35,2 °C. Pulse in radial artery is not determined.
Tachypnea, muscle spasm, urination is absent for 6 hours. What is the state of
A. the patient? of IV degree
Dehydration
B. Dehydration of I degree
C. Infectious-toxic shock
D. Anaphylactic shock

E. * Uncompensated hypovolemic shock


136. Sick P., 25 years old, presents with frequent vomitting. Objectively: dryness of
skin and mucous membranes, brief cramps in gastrocnemius muscles, the
temperature of body is normal, voice is hoarsed, moderate tachycardia and
hypotension. The compensated metabolic acidosis is marked. About what
A. degree of dehydration is it possible to think?
IV
B. II
C. * III
D. I
E. There is no dehydration
137. Patient C, 17 years old, became ill suddenly. Profuse diarrhea with frequent
vomits without nosier. A patient arrived from south-east Asia. Temperature –
36,1 °C. An abdomen is pulled in, painless. The stool has the appearance of rice-
water. What diagnosis is most reliable?
A. * Cholera
B. Dysentery
C. Salmonellosis
D. Esheryhiosis
E. Rotavirus gastroenteritis
138. A patient L., who returned from Crimea, developed diarrhea at 5AM. Bowel
movements are each 1-1.5 hrs, watery, without mucus and blood. In 12 hrs a
single episode of vomiting developed. The temperature of body at first rise to
37.3 °C, stomach-aches is present. he was examined by the doctor of first-aid
and delivered to an infectious isolation with the diagnosis of acute intestinal
A. Intestinal echerihiosis
B. Salmonellosis
C. Echeriosis
D. Food poisoning
E. * Cholera
139. The disease began sharply from diarrhea, that was accompanied by an
abdominal pain. The act of defecation brought facilitation. Emptying of green
color, abundant, foamy, with a strong unpleasant smell. Temperature of body is
subfebrile. It is found out hyperemia and graininess of soft palate. In blood:
A. leucopenia, eozinophilia. Violations of water-electrolyte balance are
Cholera
B. Toxic food-borne infection
C. Salmonelliosis
D. Dysentery
E. * Rotaviral gastroenteritis
140. Patient of 20 years old, on the 2nd day of illness painful swallowing, general
weakness, headache complaints about. Objectively: temperature 39.0 °C, pulse
– 110 per 1 min, mucous of mouth is hyperemic, tonsils is enlarged in sizes,
loose, covered by the raid, that is easily taken off, surface after the removal is
A. not bleeding. What disease is it necessary to think about?
Tularemia
B. Diphtheria of mouth
C. Simanovsky-Vensent’s tonsillitis
D. Infectious mononucleosis
E. * Acute streptococcus tonsillitis

141. Sick F., 16 years old, complaints about a general weakness, painfull swallowing,
pharyngalgia. Objectively: the mucous membrane of mouth is brightly red,
tonsils are covered by white raids, which were taken off easily, increasing of all
groups of lymphonodes, 1-3 cm in diameter, dense, elastic, not soldered
between itself. A liver is megascopic on 3 cm, spleen – on 1 Cm. Common blood
A. * Infectious mononucleosis
B. Acute lympholeukosis
C. Acute streptococcus tonsillitis
D. Diphtheria
E. Adenovirus infection
142. Patient B., 18 years old, entered infectious department with complaints about
headache, general weakness increasing of temperature to 37.5-38.2 °C during 6
days, pharyngalgia. Objectively: all groups of lymphonodes are megascopic 1-3
cm in a diameter, elastic, littlesickly, not soldered between itself. A liver is
megascopic on 3 cm, spleen – on 1 cm. Common blood analysis: atypical
A. Acute lympholeukosis
B. Adenoviral infection
C. * Infectious mononucleosis
D. Diphtheria
E. Acute streptococcus tonsillitis
143. At a patient which carried diphtheria with an incessant pharyngalgia, disartria,
weakness and loss of motions; hyperreflexia, decline of sensitiveness on a
A. polyneuritis
Hemorrhagetype developed. Put a diagnose.
in a brain
B. Viral encephalitis
C. * Diphtheria polyneuropathy
D. Psevdobulbar syndrome
E. Bulbar form of lateral Amiotrophic sclerosis
144. A patient X., 25 years old, was examined by a otolaryngologist on the 4th day of
illness. Temperature of body – 38.1 °C. Complaints about indisposition, moderate
pains in the throat. Objectively: a mouth opens fully. Mucous soft palate,
handles, tongue, was swollen, insignificant hyperemia with cyanosis. Increasing
of tonsills, covered by the grey dense raid. The raid is taken off free. The edema
A. Lacunar tonsillitis
B. Infectious mononucleosis
C. Simanovsky-Vensent’s tonsillitis
D. * Diphtheria of oropharynx
E. Follicular tonsillitis
145. At sick person with the combined form of diphtheria of tonsills, larynx and front
departments of nose on the 2nd day of stay in permanent establishment the
state became worse: a weakness grew, pain appeared after a breastbone,
vomiting. Objectively: patient is adinamic, pale, acrocyanosys. Pulse 54 per 1
min, weak filling. Cardiac tones are muffled, chaotic extrasystoles. BP is 80/40
A. Infectiously-toxic shock
B. Anaphilaxic shock
C. Cardiogenic shock
D. * Early myocarditis
E. Late myocarditis

146. Child 3 years from having many children social-unprosperity family is ill 3 days.
The disease began sharply, from the increase of temperature to 38,0 °C, barking
cough hoarseness voice. In a dynamics on a background of subfebrile
temperature a cough became soundless. At a review: the state is heavy, inciter
shortness of breath with the indrawing of jugular fossula, cyanosys of nasolabial
triangle, HR –160 per minute, tonsills filling out, hyperaemia accented
A. Viral croup
B. * Diphtherial croup
C. Respirator chlamidiosis
D. Infectious mononucleosis
E. Epiglottiditis
147. A diagnosis is “diphtheria of otopharynx” put to the 3-years-old child (not
instiled through the refusal of parents). Family lives in a 3-room apartment, a
mother does not work, a father is the director of factory. Specify the method of
isolation of hospitalization
A. * Obligatory child.
B. Hospitalization after clinical testimonies
C. Hospitalization after epidemiologys testimonies
D. Isolation in home terms
E. Does not need isolation
148. Sick, 49 years old, became ill 3 days ago: fever 37,2 °C, pharyngalgia. On right
tonsill grey pellicle for a gum-blush by a thickness 2-3 mm, spreads on a soft
palate. Previous diagnosis: Diphtheria of right tonsill and soft palate. What
method of laboratory
A. * Bacteriologic research
examination most
to the effective
stroke, takenin this case?
under pellicle
B. Microscopic research to the stroke, taken from under pellicle
C. Immunological hemanalysis
D. Biological method of research
E. Biochemical hemanalysis
149. Sick, 17 years old, became ill sharply after supercooling: fever 39,5 °C, pain of
head and muscles, pharyngalgia at swallowing, aching pain at lumbar region,
palpitation. Palate and tonsils are hypertrophied, bright red, in the lacunes
festering raid. A lacunar quinsy is diagnosed. What method of laboratory
A. research most
Biochemical effective in this case?
hemanalysis
B. Microscopic research of festering maintenance of lacunes
C. Immunological hemanalysis
D. Biological method of research
E. * Bacteriologic examination of festering maintenance of lacunes
150. Patient B., 22 years, became ill sharply, when insignificant pharyngalgias
appeared at swallowing, nasal voice. Objectively: on left tonsill and there is the
pellicle raid on a handle. It is diagnosed: lacunar quinsy, appointed penicillinum
and rinse 2 % by solution of soda. On the second day pellicles spread on a soft
palate and tongue. On a neck the edema appeared to the collar-bone, voice
A. Antibiotics of group of penicillinum and diphtherial toxoid
B. By the rinse 2 % by soda solution in combination with the peroral setting of
C. erythromicinum
Autohemotherapy, warming compresses and quartz
D. Opening paratonsill cellulose
E. * Antitoxic antidiphterial whey and benzylpenicillinum
151. Diagnostics of quinsy background on such criteria: epidemiologys information,
sharp beginning, fever, tonsillitis, regional lymphadenitis, neutrophilic
leycocytosys. What research must be conducted in an obligatory order by a
A. patient with
Selection themucus
from diagnosis of quinsy? haemolitic streptococcus
of otopharynx

B. Biochemical blood test


C. * Strokes from otopharynx and nose on bacteria of diphtheria
D. Hemoculture
E. IFA
152. A 19-years-old patient became ill 5 days ago: subfebrile temperature, acquired
voice, dry cough weakness. After hospitalization: t – 37,1 °C, but the state is
heavy, pallor, expressed weakness, aphonia, noisy stenotic breathing, inciters
indrawings of intercostal intervals, pulse-110 shots per min., frequency of
breathings 36 per min., cyanosys of lips and nailes. Tonsills is covered by the
A. Tracheotomy, AVL, antitoxin
B. Tracheotomy, AVL, toxoid, desintoxic therapy
C. * Sedative, intubation, antitoxin, dexametazon, desintoxic therapy
D. Intubation, sedative, toxoid, penicillinum, desintoxic therapy
E. Sedative, oxygenotherapy, antitoxin i/v
153. A girl, 18 years old, became ill sharply: indisposition, pain of head, pharyngalgia,
T – 38,2 °C. To the doctor appealed only on the 3th day of illness. Objectively: on
the surface of one tonsill the crateriform ulcer covered by the gray raid. During
the direct bacterioscopy exciters morphologically sticks similar to spirohets was
A. found. What
Aphthosis is previous diagnosis?
stomatitis
B. Diphtheria
C. * Vensan tonsillitis
D. Herpetic quinsy
E. Infectious mononucleosis
154. At the patient P, 20 years old, reconvalescent from heavy form of diphtheria
after 10 days of treatment a general weakn pain and paresthesias appeared (5th
week from the beginning of disease), mainly in the distal part of extremities.
Independently can not be tricked into feet. Objectively: considerable symmetric
atrophy of muscles of extremities, especially – lower. To the tendon reflexes are
A. * Syndrome of Gienne-Barre
B. Diphtherial polyneuropathy
C. Poliomyelitis
D. Transversal myelities
E. Ascending paralysis of Landri
155. Patient was consulted by doctor with complaints on sore throat. Objective status:
subfebrile temperature, moderate hyperemia of pharynx with cyanotic tint, left
tonsil is swollen with thick gray-white layer, which is hardly removed with
bleeding. Regional submandibular lymph node is increased, light painful. What is
A. preliminary
Lacunar diagnosis?
tonsillitis
B. Plaut-Vensan’s tonsillitis
C. Adenoviral infection
D. * Localized diphtheria of pharynx
E. Infectious mononucleosis
156. Patient B, after disease, which was accompanied by fever and sore throat, has
signs of dysphagia, dysarthria, weakness of limb muscles, hypoaesthesia of
polyneurotic type. What is the most possible diagnosis?
A. Sublingual nerve neuropathy
B. * Dyphtherial polyneuropathy
C. Glossopharyngeal nerve neuropathy
D. Encephalitis

E. Bulbar syndrome
157. Male patient, 22 years old, complains of sore throat, general weakness,
headache. Objective status: fever 38 °C), Ps 100 per min, pharyngeal mucosa is
hyperemic, tonsils are swollen, porous, covered with layer, which is easily
A. removed with
Follicular spatula, without bleeding. What is the most possible diagnosis?
tonsillitis
B. * Lacunar tonsillitis
C. Simanovsky-Vensan’s tonsillitis
D. Infectious mononucleosis
E. Localized dyphtheria of pharynx
158. 18 year old student was hospitalized to the infectious disease department on the
2nd day of disease with complaints on sore throat during swallowing. Objective
status: t – 37,5 °C, light hyperemia and swelling of right tonsil, which is covered
by gray-white color pellicle, slightly erected above the tonsill’s surface. The
pellicle is easily removed by spatula, disclosing hemorrhagic ulcer with smooth
A. Diphtheria of pharynx.
B. * Plaut-Vensan’s tonsillitis
C. Lacunar tonsillitis
D. Follicular tonsillitis
E. Ulcerative-necrotic tonsillitis
159. A new born child on 10th day of life became worse: Temperature 39.2 °C,
vomiting, generalized cramps, violations of consciousness, spastic paresis of left
extremities. Month prior to his birth herpes virus was present in the mother, she
A. * did not treat.
Herpetic What disease is most possible?
encephalitis
B. Meningococcal meningitis
C. Subarechnoid hemorrhage
D. Cerebral abscess
E. Violation of blood cerebral circulation
160. A patient, 22 years old, became ill sharply. History showed fever up to 38.2 °C
with headache, repeated vomiting, olfactory and tastes hallucinations. Quickly
got complex of meningeal symptoms, pyramidal paresis. The general epileptic
attack and comatose state also developed. Neurolymph is with mixed
A. lymphocytosis,
Brain abscess cytochrome, single red corpuscles. What is previous diagnosis?
B. Subdural empyema
C. * Herpetic encephalitis
D. Tumor of brain
E. Encephalopathy
161. In a newborn, the septic state was accompanied with the increase of
temperature to 40 °C) which is developed on 5th day, with pneumonia
expressed intoxication: pallor, vomiting, shortness of breath, disturbance,
cramps. Rash also appeared (blisters with hemorrhages) on skin, mucous
membrane of mouth cavity, throat and conjuctiva. The child suffers with
A. * Herpetic infection
B. Cytomegaloviral infection
C. Chicken pox
D. Rubella
E. AIDS
162. A boy 11 years old, complaints about sickness at mastication, increasing of
temperature to 37.1 °C, enlargment of parotid salivary glands . At the age of 8
years carried a paraflu infection. Objectively: in the region of right parotid
salivary gland tubular sickly at palpation, a skin above it is not changed. A
pharynx is moderate hyperemic, tonsils are not coated. What is your previous

A. * Cytomegaloviral syaloadenitis
B. Lymphadenitis
C. Parotitis
D. Infectious mononucleosis
E. Cholylithiasis
163. A woman 65 years old the disease had beginning sharply from increase of
temperature to 39.0 °C, weakness, and pain in the left part of thorax that
increased with breathing motions. On 3rd day of disease vesicular breathing
appeared after motion of rib on the left on a hyperemic background. Together
with sick a grandchild lives 4 years. What measures of prophylaxis of disease
A. Vaccination
B. Final disinfection
C. Reception of specific immune globulin
D. * Isolation of patient
E. Acyclovir administration
164. The patient, 58 years old, was hospitalised in the infectious department with
complaints of pain in the left half of thorax, fever. At a review: temperature of
body 37.5 °C, in XI-XII intercostal area the grouped shallow blisters on a
hyperemic-filling background are filled by transparent maintenance. Preparation
A. of choice for treatment of this patient is:
Suprastin
B. Prednisolone
C. Biseptolum-480
D. Semavin
E. * Laferon
165. A 40 years old scientist, became ill sharply with chill, temperature of body – 39.8
°C, severe headache, vomiting, pain in muscles. Did not went to the doctor, the
state had become worse, hyperemia of sclera, appeared, on lips herpes with
hemorrhagic maintenance, the nose-bleeding, skin and sclera, became icteric,
urine color is of strong tea, diuresis 200 ml, an anacholia was not present. What
A. * Rats
B. Cats
C. Infected people
D. Bacillocarrier
E. Mosquito
166. To the infectious diseases department was admitted patient M. 58 years old,
with complains of pain in left part of thorax, fever. During a review: body
temperature 37,5 °C, on the level of XI–XII intercostals spaces on hyperemic-
edemous shadow grouped small vesicles with transparent content. Preparation
A. of choice for treatment of this patient?
Suprastyn
B. Prednisolon
C. Biseptol-480
D. Cymeven
E. * Laferon
167. A patient came with complaints of sickly erosions on his penis. From anamnesis
frequent appearance of similar rashes is found out during a year. Objectively: on
a balanus are the grouped blisters and erosions, soft on palpation. What is your
A. * diagnose?
Recurrent herpes of ²² type
B. Vulvar pemphigus
C. Primary syphyllis

D. Shankoform pyoderma
E. Recurrent herpes of ²²² type
168. Patient A., complains of redness of the skin and edema on the right cheek.
During a review: body temperature 38,7 °C, enlarged and painful right
submandibular lymphatic nodes, the border between red and normal skin is
sharp, present vesicles with dark content, palpation is painful. Your primary
A. * diagnosis? hemorrhagic form
Erysipelas,
B. Anthrax, skin form
C. Herpetic infection
D. Varicella (chickenpox)
E. Phlegmone of the cheek
169. In a policlinic the patient C., 18 years old, appealed with complaints about a
moderate pharyngalgia, head pain, general weakness fervescence, to 38,9 °C. It
is ill already three days. Objectively: it is found out megascopic back neck, and
inguinal lymphatic knots, hepatospleenomegaly, subicterus scleroticas and skin,
changes in a throat, characteristic for tonsils. At the hemanalysis –
A. * Infectious mononucleosis
B. Follicle quinsy
C. Adenoviral infection
D. Flu
E. Viral hepatitis
170. Patient D., 32 years, appealed on the third day with complaints about a sharp
general weakness, change of voice, pharyngalgia. Objectively: dense pellicles
occupy all surface of tonsills, tongue, handles; a hypoderm was swollen from
both sides to the middle of neck. Cardiac activity is satisfactory: pulse 90 per
A. min, BP is 95/65mm
Combined diphtheriaHg; temperature of body 38,3 °C. Your diagnosis?
B. Diphtheria of otopharynx, moderateform
C. Infectious mononucleosis, severy form
D. * Diphtheria of otopharynx, severy form
E. Diphtheria of nasopharynx, severy form
171. The patients, 20 years old, with 5 days fever, pain in the throat during
swallowing, pain in muscles. During review – there is thick, congested tonsils
with purulent layers that are easily removed, enlarged neck, and inframaxillary
lymph glands, liver + 1.5 cm possible palpation of edge of spleen. What
A. additionalofresearch
Puncture should
the lymph be followed
nodes, done? by microscopy
B. Ultrasound of the abdominal cavity
C. * Blood test for atypical mononuclear
D. Inoculation of blood
E. Reaction of passive heamoglutination(RPH) influenza A viruses
172. At a patient with meningococcal meningitis 44 years old, rose up general clonic-
tonic cramps, abundant sweat, hyperemia of person, bradycardia quickly
changed on tachycardia, violation of breathing (Cheyne-Stokes type). What
A. complication
Infectious develop?
toxic shock
B. * Wedging of cerebrum in the cervical channel
C. Encephalitis
D. Swelling and edema of cerebrum
E. Waterhause-Friedrichsen syndrome
173. Patient D., 20 years old, fell ill and went to infectious polyclinic cabinet with
complaints of moderate pain in the throat, headache, general weakness,
increased body temperature of 38.9 °C. Fells bed during three days. At
examination: increasing of inguinal lymph nodes, hepatosplenomegaly,
subicteric sclera and skin, changes in the throat are typical for sore throats. In
A. * Antibiotics, interferons, hepatoprotectors
B. Antihypertensives hepatoprotectors, antihistamines
C. Antibiotics, hepatoprotectors, antihistamines
D. Antibiotics, antihypertensives, antihistamine
E. Antibiotics, interferones, vitamins
174. 35 years old patient A., came to the clinic on the third day of illness with
complaints of drowsiness, sweat, headache, fever up to 38.5 °C. Reviewing
physician found sore throat, splenomegaly, enlarged lymph nodes. Liver is not
enlarged. Blood analysis – leukocytosis, lymphocytosis. What additional test
A. should hold
Byurne the patient for infectious
and Wright-Hadulson’s mononucleosis?
reaction
B. ELISA-test, bacteriological test for tularemia
C. Bacteriological test for diphtheria and typhoid fever
D. * ELISA-test, bacteriological test for diphtheria
E. Paul-Bunnel’s reaction and lymph node puncture
175. At a patient with meningococcal meningitis, in 52, there was the syndrome of
cerebral hypotension on the 6th day of illness. Objectively: expressed toxicosis
and dehydration of organism, meningial symptoms disappeared, muscles tones
A. * was low and
Infectious areflexia
toxic shock developed. What complication arose up?
B. Wedging of cerebrum in the cervical channel
C. Encephalitis
D. Swelling and edema of cerebrum
E. Waterhause-Friedrichsen syndrome
176. Patient D., 30 years, have been examined by emergency doctor and delivered in
a clinic on the 2th day of illness. Fell ill sharply, temperature 40 °C, sharp head
acke, repeated vomits, photophobia. The common state is severe, consciousness
is darkened. Star-like shape rashes at the skin of abdomen, buttocks and lower
extremities appeared. Sharply expressed rigidity of cervical muscles, positive
A. * Meningococcal infection
B. Flu, toxic form
C. Marburg hemorrhagic fever
D. Leptospirosis
E. Hemorrhagic stroke
177. Patient Í., 21 year, entered hospital on the 7th day of disease with complaints
about the expressed weakness, pains in muscles and joints, head pain, nausea.
The disease began with irritation on the throat, chill, general weakness. After 2
days the state became worse, temperature increased to 39-40 °C. Objectively:
patient adynamic, consciousness is stored. Temperature of body 37,5 °C, the
state as severe, pale skin, lips and nail phalanxes cyantic, hemorrhagic rashes
on the skin of trunk and hands, the scopes of heart are extended to the left on
1,5 cm, tones deaf, pulse 130/min, weak filling, BP 80/40 mm/Hg. Vesicular
A. Hemorrhagic fever
B. Leptospirosis
C. Epidemic typhus
D. * Meningococcal infection
E. Typhoid fever
178. Patient B., 38 years, delivered by emergency doctor, anamnesis is unknown.
However, according to neighbours, patient 2 days ago was healthy. At a review:
consciousness is absent, motion of left overhead and lower extremities is
absent, increasing of muscular tone is marked in the same extremities.
Periodically are clonic-tonic cramps in right extremities. Expressed asymmetry of
person. Sharply expressed rigidity of muscles of the back of head, positive
Êårnig symptom, positive overhead and lower Brudzinsky symptom. On the skin
of overhead and lower extremities, buttocks, trunk abundant hemorrhagic
rashes “star shape” character with necrosis in a center. Acrocyanîsis is marked,
temperature of body 38,8 °C, hyperemia of pharynx Breathning 36/min,
unrhythmical. Vesicular breathing during auscultation. Pulse 72/min, weak filling
A. Epidemic typhys
B. Hemorrhagic fever
C. Leptospirosis
D. Typhoid fever
E. * Meningococcal infection
179. At a patient with meningococcal nasopharynsitis in 2 days from the beginning of
illness the temperature of body rose to 41 °C, head pain, vomit, positive Kernig
and Brudzinsky symptoms appeared. In cerebrospinal liquid: cytosis 15 000 in 1
A. mkl, 90 %
Chronic are neutrophils. What diagnosis?
meningococcemia
B. * Meningitis
C. Endocarditis
D. Infectious-toxic shock
E. Meningism
180. At a patient with nasopharyngitis after 2 days from the beginning of illness the
temperature of body rose up to 41 °C, head pain, vomit, positive Kernig and
Brudzinsky symptoms appeared. Cerebrospinal liquid: cytosis 15000 in 1 mkl, 90
A. % are neutrophils.
Chronic Diagnosis?
meningococcemia
B. Endocarditis
C. * Meningitis
D. Meningism
E. Meningococcal nasopharyngitis
181. A patient was delivered in permanent establishment on the 2th day of illness
with the symptoms of expressed toxicosis. Objectively: found out the severe
condition, hemorrhagic rash of wrong form of different size on a skin,
hemorrhages in the scleras of both eyes. Extremities are cold. AP 60/20 mm/hg,
A. * pulse – 120 per minute. Previous diagnosis?
Meningococcemia
B. Hemorrhagic fever with a kidney syndrome
C. Leptospirosis
D. Typhoid fever
E. Pseudotuberculosis
182. A patient sharply had a chill, head pain, vomits, temperature of body rose up to
38,5 °C. Till evening rigidity of muscles, Kernig symptom appeared. Herpetic
blisters are marked on mucouse of lips and nose. Neurological symptoms is not
A. found out expressed.
Subarachnoiditis What disease will you suspect?
hemorrhage
B. Herpetic encephalitis
C. Abscess of brain
D. Hemorrhage in a brain
E. * Meningococcal meningitis

183. Patient A., 23 years suddenly experienced intensive head pain, nausea, pain in a
neck, and lumbar. Examination is delivered in a clinic. Objectively: hemorrhagic
rashes on a body. Temperature 40,2 °C. Expressed meningeal symptoms. Light,
haptic, pain hypesthesia. Blood: leucocytes – 25 thousand/l, ERS-29 mm/hr. A
neurolymph is turbid, neutrophilic erythrocytosis, meningococci. What is your
A. Staphylococcus meningitis
B. Tubercular meningitis
C. * Meningococcal meningitis
D. Viral meningitis
E. Pneumoccocal meningitis
184. A patient became ill sharply. Severe head acke, frequent vomits appeared in the
morning. Temperature of body rose to 39,9 °C. Adopted febrifuge, however much
the state got worse. Till the evening patient lost consciousness. Excited, sharply
A. expressed
Status meningeal
typhosus signs.fever
in typhoid What is most reliable diagnosis?
B. Viral meningoencephalitis
C. Sepsis, infectious-toxic shock
D. * Meningococcal infection, meningitis
E. Status typhosus in epidemic typhus
185. At a patient, 35 years, sharp beginning of disease happened. Symptoms of
nasopharyngitis. Body temperature – 39,5 °C, head ache, which is accompanied
with vomits, myalgias. Hemorrhagic rash, that are localized on scleras, brushes,
feet, buttocks. Rigidity of cervical muscles, Kernig and Brudzinsky symptoms are
expressed. Neutrophilic leucocytosis in common blood analisis.What is most
A. * Meningococcal infection
B. Typhoid fever
C. Yersiniosis
D. Leptospirosis
E. Epidemic typhus
186. Person 16 years entered permanent establishment on 6 day of illness. Illness
began from a cold and cough. Temperature 37,7 °C. The state became worse:
severe head pain, frequent vomits unconnected with a meal appeared. The
common state is heavy. Pupils are extended, photoharmose is not present.
Positive meningeal symptoms. General hyperesthesia. Tones of heart are deaf,
A. Infectious mononucleosis
B. * Meningococcal meningitis
C. Toxic food-born infection
D. Influenza
E. Typhoid fever
187. A 17 years old boy, got ill suddenly: the temperature rose up-to 40,3 °C,
extremely sharp headache, makes him to yell. The patient is agitated and
vomits frequently. Tremor of fingers of extremities is observed. The star like
hemorrhagic rash of different form and sizes appeared, mainly on buttocks,
A. thighs, shins, and trunk. Meningeal signs are positive. What is the most credible
Encephalitis
B. Flu with a hemorrhagic syndrome
C. * Meningococcal infection
D. Measles
E. Leptospirosis
188. Sick 15 year old, became ill 3 days ago. Disturbed| headache, running nose,
pharyngalgia, temperature of 37,4 °C. Next days: headache increased, a
temperature had rose to 38,3 °C, repeated vomiting. Objectively: a skin is pale,
without rash. Mucus of the posterior wall of pharyx moderate bloodshed filling
out. Pulse 86/min satisfactory filling. Meningeal signs are positive. It is needed to
A. Bioassay on mice
B. Endermic test
C. Widal test
D. Cultivation on bilious clear soup
E. * Bacteriological examination of CSF
189. Sick, 54, hospitalized in an infectious department in a severy condition.
Complaint about the expressed headache, mainly in frontal and temporal areas,
superciliary arcs, origin of vomit on height of pain, pain at motions by eyeballs,
in muscles and joints. Objectively: a patient is excited, temperature of body 39
°C, BP100/60 mm Hg. Bradycardia changed by tachycardia. There were tonic
cramps. Doubtful meningeal signs. From anamnesis it is known that at home a
A. * Mannitol, lasix, prednisolone, euphylin, suprastin
B. Mannitol, acetophene
C. Lasix, analgin, ampicillin
D. Veroshpiron, euphylin, dimedrol
E. Aspirin, analgin, dimedrol
190. A patient, 20 years old, during few days complains about pharyngalgias. After
supercooling the state became worse : sudden chills, increase of temperature to
40,6 °C, headache. On skin of lower extremities trunk and buttocks there are a
lot of different sizes of hemorragic spots, acrocyanosis. Consiouness is
A. * preserved.
MeningococcalMeningeal signs are absent. What is the previous diagnosis?
infection
B. Flu
C. Epidemic typhus
D. Hemorrhagic fever
E. Leptospirosis
191. A patient C., 25 years old, fell suddenly ill. Every morning severe headache,
frequent vomiting, temperature of the body is 39,9 °C. Adopted fatigue, then
state got much worse. In the evening lost of consciousness. Expressed muscles
pains of back and head. Positive Êårning’s symptom. Leukocytes – 18,0*109.
A. What is the most reliable diagnosis?
Flu
B. Epidemic typhus, typhus state
C. Viral menigoencephalitis
D. Sepsis, infectious-toxic shock
E. * Bacterial menigoencephalitis
192. A patient 17 years old, became suddenly ill: .Temperature rose to 40,3 °C.
Severe headache, motive excitation, frequent vomiting, tremor of fingers of
extremities. Hemorrhagic spots of round form and different sizes, more
frequently as stars, mainly on buttocks and trunk. Meningeal signs are positive.
A. What is the most possible diagnosis?
Encephalitis
B. Flu with a hemorrhagic syndrome
C. * Meningococcal infection
D. Measles
E. Leptospirosis
193. In permanent establishment on the 2nd day of illness a patient is delivered with
the symptoms of expressed toxicosis. Objectively: the general condition is
severe, on skin hemorrhagic rash of different form and sizes, hemorrhages in
sclera of both eyes. Extremities cold. AP is 60\20 mmHg., pulse – 120. Previous
A. diagnosis?
Meningococcemia
B. Crimean hemorrhagic fever
C. Hemorrhagic fever with a renal syndrome
D. Typhoid fever
E. ECÍÎ-eczanthema
194. Patient, 16 y.o., during 3 days there was a increase of temperature to 38 °C,
ïåðø³ííÿ in a throat, indisposition. Next 2 days felt better. A temperature was
normal. There was a chill on a 6th day, a temperature rose to 40 °C, intensive,
quickly increased headache, through 3 hours – repeated vomits. The state is
heavy, consciousness is absent, psychomotor violations expressed meningeal
A. Tubercular
B. Enteroviral
C. Postinfluenzal
D. * Meningococcal
E. Lymphocytic choreomeningitis
195. Patient D., 30 y.o., fell ill sharply, temperature 40 °C, severe headache, repeated
vomits, photophobia. The general condition is severe, consciousness is
darkened. On the skin of abdomen, buttocks and lower extremities a lot of
starlike rash elements. Expressed rigidity of neck muscles, positive Kernig
symptom are present. Pulse 100 per 1 min, weak; deaf heart tones , AP 60/30
A. Flu, toxic form
B. Crimean hemorrhagic fever
C. Disease of blood
D. Hemorrhagic stroke
E. * Meningococcal infection, meningococcemia
196. Worker, 22 y.o., became ill sharply: t 39°C, great pain of head, frequent vomits.
Objectively: the condition is severe, psychomotor excitation, moans due to the
great pain of head, expressed rigidity of neck muscles. Sharply positive
symptoms of Brudzinsky’s and Kernig’s, general hyperesthesia. What needs to
A. * be done blood
General for clarification
analysis andof diagnosis?
liquor analysis
B. General blood analysis and blood sterility
C. Computer tomography of cerebrum
D. X-ray of skull in two projections
E. Analysis of the excrement and urine for a pathological flora
197. Patient K., 21 y.o.., the disease began from the increase of temperature to
39,0°C, headache, chill, repeated vomit. Objectively: temperature of 39,3°C,
pulse 76 tense. Rigidity of muscles of the back of head. The tendon reflexes are
expressed. Analysis of liquor: cytosis 1237 in 1 ml, from them: 84 %
neutrophiles, 16 % lymphocytes, reaction to Panda ++, albumen 0,66 g/l, liquid
is turbid, flows out under the promoted pressure. Bacterioscopy found out in
A. Serous meningitis.
B. Infectious mononucleosis
C. * Meningococcal infection:purulent meningitis
D. Second festering meningitis
E. Meningococcal infection: serous meningitis

198. In a patient, 27 y.o., on the 5th day of respirator disease, there was sharp head
pain, that was accompanied by nausea, repeated vomits, hyperesthesia,
photophobia. At a review: lies with the neglected head, feet are bended to the
trunk, expressed rigidity of muscles of the back of head, positive symptoms of
Kerning’s, Brodzinsky’s. It is not found out paresises. Select a basic neurological
A. * Meningeal syndrome
B. Syndrome of liquor hypertension
C. Syndrome of liquor hypotension
D. Root syndrome
E. Vegetative crisis
199. Patient with intensive head acke, nausea, pain in a neck and lumbar area by
emergency group was delivered to the clinic. Objectively: on a body
hemorrhagoc rash.. Temperature of 39,2 °C. Expressed meningeal symptoms.
Light, tactile, pain hyperesthesia. Blood: leucocytes-25x10 9/l, ESR-29 mm/hour.
A. * What method
Lumbar punctureof inspection is most informing?
B. Computer tomography
C. Electroencephalography
D. Transcranial dopplerography
E. Echoencephalography
200. Patient, 15 y.o., became ill 3 days ago. Complains of headache, çàêëàäåí³ñòü
nose, pain in a throat, temperature of 37,4 °C. Next days : headache increased,
a temperature risen to 38,3°C, repeated vomits. Objectively: a skin is pale,
without rash. Mucous membrane of posterior wall of throat is moderately
hyperemic, swollen. Pulse 86, satisfactory filling. Meningeal signs are positive.
A. * Bacteriological research of liquor
B. Biological test on mice
C. Intracutaneous test
D. Reaction of Vidala’s
E. Smear of the blood on bilious bullion
201. In a patient, 35 y.o., sharp beginning of disease, precedingnasopharyngitis, t°
39,5°C, headache, which is accompanied with vomits, myalgias, hemorrhagic
rash, that is localized on sclera, fists, feet, buttocks; rigidity of cervical muscles,
symptoms of Kering’s and Brudzinsky’s are positive, expressed neutrophilic
A. leukocytosis.
Typhoid fever What most credible diagnosis?
B. Yersiniosis
C. Leptospirosis
D. * Meningococcal infection
E. Thrombocytopenic purpura
202. Sick C., 8 years, appealed to the infectious hospital on the second day of disease
with complaints about a pharyngalgia at swallowing, increase of temperature.
Objectively: temperature 38,6 °C, sharp hyperemia of soft palate, tonsills, filling
out, loose, on both there are festering stratifications which are taken off by a
spatula. Megascopic sickly submandibular lymphonoduses are palpated. Pulse –
114 per a min. Present roseol-papular pouring out on all body. Pastia‘s symptom
is positive. It is known from epidemogical anamnesis, that its comrade had alike
A. Lacunar quinsy
B. Diphtheria of pharynx
C. * Scarlet fever
D. Flu
E. Infectious mononucleosis

203. A 4 years old child complains about: cough, temperature of body 38,1 °C. Skin
without rashes. Conjunctiva hyperemic. Exanthema on skin. On mucous of
cheeks there are points of hyperemia gum blushs. In lungs difficult breathing.
A. What is fever
Scarlet the most possible diagnosis?
B. Rubella
C. * Measles
D. Enteroviral infection
E. Flu
204. Sick C., 8 years, appealed to the infectious hospital on the second day of disease
with complaints about a pharyngalgia at swallowing, increase of temperature.
Objectively: temperature of body – 38,6 °C, sharp hyperemia of soft palate,
tonsills, filling out, loose, on both there are festering stratifications which are
taken off by a spatula. Palpated megascopic, sickly submandibular
lymphonoduses. Pulse – 114 per min. Present roseol-papular pouring out on all
body. Pastia symptom is positive. It is known from epid anamnesis, that at its
A. lacunar quinsy
B. Diphtheria of pharynx
C. Infectious mononucleosis
D. Flu
E. * Scarlet fever
205. Patient A 24y/o. Appealed on the 5th day of illness with complaints about cough,
malaise, fever up-to 38,1 °C. On face and behind the ears maculo-papular rash,
appeared a day ago, and other areas of skin are without rash. Conjunctiva is
hyperemic. Enantema on a soft palate, on the mucous membrane of cheeks are
whitish points with the halo of hyperemia. There is the hard breathing in the
A. Scarlet fever
B. German measles
C. * Measles
D. Enteroviral infection
E. Flu
206. A patient was seen by a doctor on the 2nd day of illness with complaints about a
fever up-to 38,1 °C, painful swallowing, malaise. On the skin of trunk and
extremities, especially in natural folds, point rash on hyperemic background was
faund. The nasolabial triangle was white with no rash. Enantema on a soft
palate, and purulent exudates on tonsils were observed. A few enlarged and
A. * Scarlet fever
B. Rubella
C. Measles
D. Enteroviral infection
E. Flu
207. A 32.y/o man was examed by a doctor with complaints about subfebrile
temperature, and insignificant catarrhal phenomena. On the skin of trunk and
extremities spotted-papular elements of rash without a tendency to congregate
was observed. The mucous membrane of oro-pharinx was not changed. A few
A. enlarged
Scarlet and painful cervical lymph nodes were found. What is the diagnosis?
fever
B. * Rubella
C. Measles
D. Enteroviral infection
E. Flu

208. A 24 y/o patient was seen by a doctor on the 2nd day of illness with complaints
about a fever up-to 38 °C, headache, and malaise. On the skin of hairy part of
head, runk and extremities vesicular rash with red hallow was found. Also some
vesicular elements were found on the mucous membrane of pharynx. Lymph
A. nodes were
Scarlet fevernot palpable. What is the most probable diagnosis?
B. Rubella
C. Measles
D. * Chicken-pox
E. Herpetic infection
209. Patient S., 27 years old, was admited on the 4th day of illness with the diagnosis
of viral ARVI, and allergic dermatitis”. The patient felt ill suddenly with increase
of temperature up-to 38 °C, headache, sore throat, and intensive cough. On the
3rd day rash appeared on the skin of neck and face. Patient was taking aspirin in
order to decrease the temperature. Objectively: temperature of body 38,8 °C.
The face of patient looks puffy. Signs of conjunctivitis, and renitis were observed.
On the skin of neck face and chest there was intensive papular rash, without
itch. Mucous membrane of oropharynx was brightly hyperemic. Submandibular
A. * Measles
B. Allergic dermatitis
C. Infectious mononucleosis
D. Rubella
E. Scarlet fever
210. A child 10 years old presents with temperature 38 °C, renitis, conjunctivitis,
moist cough. On the mucous membrane of cheeks, lips, gums there are greyish-
white points, reminding a farina. What is the diagnosis?
A. * Measles
B. Adenoviral infection
C. ARVI
D. Enteroviral infection
E. Infectious mononucleosis
211. A patient’s temperature is 40 °C. There are olso deep and unproductive cough,
photophobia, face puffiness whitish points on the mucous membrane of cheeks
opposite molar teeth. What is the most possible diagnosis?
A. Tuberculosis
B. Meningococcemia
C. * Measles
D. Enteroviral infection
E. Staphylococcus sepsis
212. ?Pregnancy woman, fell ill suddenly after 3 weeks arrival from Turkmenistan. An
icterus, incessant vomiting, pain in right subcostal area, skin haemorrhages,
were appeared after 7-daily fever. What is most credible diagnosis:
A. * Hepatitis A
B. Hepatitis B
C. Acute fatty hepatosis of pregnant
D. Cholestatic hepatosis of pregnant
E. Cholecystitis
213. Patient with chronic viral hepatitis with development of cirrhosis, hospitalized
with a diagnosis: “bleeding from the veins of gullet”. Sharp pallor of skin covers.
BP 80/40 mm Hg, pulse 100 per 1 min, the temperature of body is normal. On a
front abdominal wall a venous net is extended. What preparation is it necessary
to begin therapy from?

A. Albumen
B. Plasma
C. Neogemodez
D. * Blood
E. Glucose
214. Patient entered surgical department with bleeding from the mouth, was
admitted to the gastroenterological department. He considered himself healthy.
20 years ago had haemotransfusion. Fibrogastroduodenoscopy shows venous
dilation of the oesophagus of the III degree. Most probable etiologic reason of
A. disease:
Autoantibodies against hepatocytes
B. Viral hepatitis A
C. X-ray irradiation
D. * Viral hepatitis B
E. Defficiency of antitrypsin
215. Weakness appeared in a patient, decrease in an appetite, began pains in the
joints of extremities, felling of weight in right subcostal region, vomiting. Urine
became dark in 12 days, and in a day – sclera and skin turned yellow. It was
discovered in an immunogram: antuIgG -HAV (+), HBsAg (+), antiIgÌ -ÍBcorAg
(+), anti-ÍCV (–), anti-ÍDV (–). To the patient 3 months back was extracted a
A. * Viral hepatitis B
B. Viral hepatitis A
C. Viral hepatitis C
D. Viral hepatitis D
E. Viral hepatitis E
216. A patient during half of the year got frequent parenterally injections concerning
bronchial asthma. There was decrease in appetite gradually, started to feel
weakness, arthralgia, dark coloured urine, the icterus of skin appeared.
Objectively: temperature of 37 °C, pulse 68/min BP 115/70 mm of Hg Liver +4
cm, spleen +1 cm, skin and sclera is yellow coloured In the general blood
analysis: amount of leucocytes: 3,6 109,among them 52 % lymphocytes, ESR 6
A. Anti-HBs antibodies
B. Anti-HAV IgM antibodies
C. * Anti-HBV antibodies
D. Anti-HCV IgG antibodies
E. Anti-HEV IgM antibodies
217. Patient complains about fatigue, decreasing of appetite, nausea, feeling of
weight in a epigastric region, bitter taste in to the mouth. Objectively: Pulse is
76/min , temperature 37,2 -°C. Skin is pale, with single vascular “asterisks”
present on thorax. Liver is enlarged with 2 cm below the costal arch . In blood:
bilirubinn :36 mmol/lt , ASAT :2,5 mmol/lt, ALAT :2,8 mmol. HBeAg, anti-HBc (–),
A. * Chronic hepatitis B, phase of integration, activity is poorly expressed
B. Chronic hepatitis B, phase of replication, activity is poorly expressed
C. Chronic hepatitis B phase of integration, high activity
D. Chronic hepatitis C, phase of replication, activity is poorly expressed
E. Chronic hepatitis B phase of replication, moderate activity
218. A complains of fatigue, decreasing of appetite, nausea, feeling of weight in a
epigastric region, bitter taste in the mouth. Objective examination : Pulse
76/min, temperature 37,2 °C. Skin is pale, single vascular star-like rash on the
thorax. Liver is 2 cm below the costal arch. In blood: bilirubin level is 36 mmol/lt,
ASAT 2,5, ALAT 2,8 . Anti-HBc (–), HBsAg and HBeAg (+). What therapeutic tactic

A. Therapy by hepato-protectors
B. * Antiviral therapy by lamivudine
C. Antiviral therapy by ribavirin
D. Therapy by corticosteriods
E. Therapy by immunostimulators
219. Drug addict, takes drugs intravenously. Has been taking drugs for 12 years
Complaints about weakness, moderate icterus, weight in right subcostal region.
The state was worsened gradually. Biochemical indexes: general bilirubin 28,2
mmol/lt; ALAT 1,0, ASAT 0,8 . Will you define a diagnostic method which it is
A. expedient
Biopsy to conduct for establishment of etiologic diagnosis?
of liver
B. * Polymerase chain reaction (PCR)
C. Enzymes of liver
D. Computer tomography
E. Immunological tests
220. Patient a lot of years used an alcohol. Complaints about nausea, liquid stools,
icterus of skin. Objectively: skin and sclera is yellow coloured, atrophy of
muscles, subfebrile temperature. Liver 3 cm below the costal arch, painful on
A. palpation
Activity ofWhat method
cytolysis of diagnostics
(AsAT, AlAT) most informing in this case?
B. Proteinogram
C. * Biopsy of liver
D. Markers of viral hepatitis
E. Violation of cellular immunity (T4,T8)
221. A patient complains about attacks of pain in right subcostal area after the
physical loading. Periodically marks more light excrement, darkening of urine.
Objectively: skin and mucous membranes high icteric. General bilirubin 36,8
mm/L, direct fraction - 26,4. Ultrasound of gall-bladder: thickness of wall 4 mm,
A. there is a lot of bile in the ducts. It is necessary to prescribe with a lytolitic
Choleretics
B. * Ursofalk(ursodesoxyhole acid)
C. Cholekinetics
D. Spasmolytics
E. Cytostatics
222. During annual inspection increasing of a liver to 4 cm was found, increasing of
bilirubin level in 2 times, AlAT in 2,5 time. He has been often sick with genital
herpes. It was discovered RNA of hepatitis C virus. What is etiologic treatment
A. * Interferons
B. Essencial phosphotides
C. Choleretics
D. Cholekinetics
E. Ursodesoksihole acids
223. After violation of diet and nervous stress the state of the patient became worse:
increasing of intensity of jaundice, pulse 110 per 1 min, BP 80/50 mm Hg,
vomiting with “coffee-grounds”, decreasing of the liver size. What complication
is it possible
A. * Acute to think about?
liver insufficiency
B. Infectious-toxic shock
C. Acute kidney insufficiency
D. Acute extrarenal insufficiency
E. Hemolytic
224. Direct bilirubin is increased , in urine there is significant increase of bilirubin and
urobilin, decreasing of stercobilin of excrements. What is the type of icterus?

A. Haemolitic
B. * Parenchymatous
C. Transport
D. Extraliver
E. Mechanical
225. Patient has been sick with chronic hepatitis. Complaints about pain in the liver,
growth retardation. Objectively: his skin is dry, erythemic, atrophy of muscles,
telangiectasis on shoulders, hyperemia of hands, abdomen is enlarged, look like
“jelly-fish head”. The liver below the costal arc on 3 cm, painful, dense, spleen
on 1 cm below the costal arch, positive symptom of fluctuation. Laboratory
investigation: hypergammaglobulinaemia, increasing of AsAT activity, AlAT is
A. Lactulose
B. * Hepatoprotectors
C. Antibiotics
D. Glucocorticoids
E. Alpha interferon
226. Drug addict, complains about dull pain in the right subcostal area, weakness,
decline of appetite, pain in joints, which disturbs during 2 weeks. At
examination: icterus of the skin and sclera, enlargement of the liver and spleen,
A. dark urine, excrement is discoloured. What is preliminary diagnosis?
HIV-infecion
B. Hepatitis B
C. Hepatitis C
D. * Toxic hepatitis
E. Hepatitis A
227. On the average 15 to 30 % of all population of the planet suffer from some
pathology of liver. Prevalence of hepatitis and cirrhosis in the European countries
is about 1 % of adults. Annually in the world there are about 2 million people
with acute viral hepatitis. What % of all cases will develop chronic form.
A. 100 %
B. 50 %
C. 25 %
D. * 10 %
E. 1%
228. A patient 20 years old, had treated himself concerning an acute respiratory
disease for 5 days, but marked no displays of respirator syndrome. Last 2 days
temperature has been normal, appetite disappeared, however, appeared pain in
epigastrium with nausea, and urine turned dark. About what illness is it possible
A. to think? B
Hepatitis
B. Infectious mononucleosis
C. Pseudotuberculosis
D. Leptospirosis
E. * Hepatitis A
229. A patient 28 years old, an injection drug addict, complaints about dull pain in
right subcostal region, weakness, decline of appetite, pain in joints. The
symptoms have been present for 2 weeks. At examination: the icterus of skin
and scleras observed. The liver and spleen are enlarged. Urine is dark,
A. excrements
Hepatitis A are discoloured. What is preliminary diagnosis?
B. * Hepatitis B
C. Chronic cholecystitis

D. Toxic hepatitis
E. HIV-infection
230. A patient C., 43 years old, has been treated for 5 years. During the last
hospitalization a liver-biopsy was conducted. Histologicaly there is bulb
dystrophy and necrosis of hepatocytes, with leucocyts infiltration, and
pericellular
A. * Alcohol liverfibrosis. What diagnosis is most credible in this case?
cirrhosis
B. Viral hepatitis B
C. Viral hepatitis C
D. Cryptogenic hepatitis
E. Fatty dystrophy of liver
231. A 19 years old patient was diagnosed with hepatitis B. After violation of diet and
nervous stress the state of patient got worse: intensity of ichterus was
increased, pulse 110/min, BP 80/50 mm Hg. Also there was vomiting by «coffee-
grounds». The size of liver has decreased, the liver on palpation was painful.
A. What complication
Infectious-toxic would you think about?
shock
B. Acute kidneys insufficiency
C. Acute adrenal insufficiency
D. Hemolitic icterus
E. * Acute hepatic insufficiency
232. Patient B., 23 years old. Objectively: skin is yellow, icterus of sclera. Pulse
66/min, BP 120/80 . Indexes of AlAT, AsAT, and Tymol tests are normal. General
billirubin is 34 mmol/l due to indirect fraction. Specify the most credible
A. diagnosis.
Chronic toxic hepatitis
B. Chronic cryptogenic hepatitis
C. Chronic hepatitis C
D. * Zhilber’s syndrome
E. Chronic B hepatitis
233. A trained nurse got sick on 16.09: general weakness, nausea, vomiting, pain in
joints appeared. 21.09 urine turned dark, and skin yellow. 21.09 she was
hospitalized. Objectively: general status satisfactory, temperature 38,2 °C,
expressed skin and mucous membranes jaundice. Liver palpated 3 cm below the
costal arc. 26.09 stomach-aches and increased sleepiness with disorientation in
time and place were appeared. 27.09 haematomas in the places of injections
also appeared. Hepatic breath was noticed. Light edema of feet and lumbar
A. Meningoencephalitis
B. * Hepatic encephalopathy
C. Acute nephrosonephritis
D. Pancreatitis
E. IDS syndrom
234. A patient B., 52 years old, for 3 month complains about nausea, periodic
vomiting, swelling of stomach, weakness, loss of weight up to 12 kg, consistency
of stool is changing. During some days consciousness is entangled, somnolence,
allolalia, general weakness are expressed. Temperature of body 37,4 °C.
Icterusis on the skin. Tremor of brushes and nystagmus is marked. Pupils are
narrowed, with a slow reaction on light, increase of tendon reflexes. Ps 112/min,
unrhythmical. BP 90/65 mm Hg. A liver is dense +2. Laboratory tests: Hb 86 g/L,
Leuc 4,2*109/L, ESR 18 mm/hour, glucose 3,4 mmol/l, general bilirubin 56,7
A. Hepatocirrhosis
B. Vilsona-Konovalov disease
C. Cancer of liver

D. * Endogenous hepatic encephalopathy


E. Meygs disease
235. A patient, 17 years, complains about a weakness, worsening of appetite,
nausea, painfull in right subcostal area, dull ache in the body, dark color of
urine, temperature to 37,8 °C. She stood in contact with sick on icterusis 1,5
months ago. Objectively: yellow colour of the skin and visible mucous
membranes. The liver is increased to 1,5 cm. Unsignificant enlargment of
A. Viral hepatitis B
B. Infectious mononucleosis
C. * Viral hepatitis A
D. Leptospirosis
E. Pseudotuberculosis
236. Before the appearance of jaundice in a patient 16 years old, during 2 days there
were an increasing of the body temperature (38,5 °C), headache, dull ache
disturbed whole body. Name the variant of pre-icteric period of viral hepatitis for
this patient?
A. * Influenza-like
B. Astenovegetative
C. Artralgic
D. Dyspeptic
E. Allergic
237. A patient T., 28 years, appilled due to worsening of common status at the
seventh day to the infectious department concerning viral hepatitis. Nausea and
vomiting were present, hepatic smell appeared, increasing of the liver sizes.
A. What biochemical
Increasing activity index does allow to suspect complication in the patient?
of AlAT
B. Increasing of bilirubin, increasing activity of AlAT
C. Increasing timol test
D. * Decreasing of protrombin, increasing of bilirubin
E. Decreasing of AlAT
238. A sick P., 54 years old, complains about dull pain in right subcostal area, bad
taste in the mouth, itching of the skin, increasing of abdomen. After the liver
puncture fatty dystrophy of hepatocytes, eccentric placing of veins in a hepatic
lobule were found. What laboratory index are most characteristic in this case?
A. Alkaline phosphotase
B. Hemodiastase
C. * Transaminases level
D. Glucose of blood
E. Creatinphosphokinase
239. Student 20 years, treated oneself on an occasion of ARVI (increasing of
temperature to 38,2 °C during 3 days). He complaints about worsening of
appetite, increasing fatigue at a normal temperature and absence of the
catarrhal phenomena of upper respiratory tracts. A doctor found out an
increasing and moderate sickliness of liver. There were cases of hepatitis A in a
A. Ultrasound scanning of the liver
B. Determination of bilirubin level of the blood
C. Determining the amount of beta-lipoproteins
D. * Determination of activity of àminotransferases of the blood
E. Immunofluorescent research of the nasal smears

240. A patient, 24, complains about pains in right subcostal area, increasing after-
meal, nausea, increase temperature of body to 37,7 °C, icterus, pain in large
joints. He is ill from 8 months. Suffers a nonspecific ulcerative colitis.
Hepatosplenomegaly. ESR 47 mm/hr, general bilirubin level is 86,1mmol/lt,
direct-42,3 mmol/lt. In blood found out antibodies against smooth muscles.
General albumen of 62 gram/lt, albumin á. 40 %, globulin. 60 %, gamma
A. Primary biliary cirrhosis
B. Zhilber syndrome
C. Viral hepatitis
D. Hemochromatosis
E. * Autoimmune hepatitis
241. A patient I., 25 years old, appealed to the internist with complaints about a
general weakness, loss of appetite, pain in the right hypochodrium. Treated
himself on an occasion of ARVI ambulatory 3 days. Became acute worsening of
fealling: nausea, pain in right hypochondrium, ochrodermia of sclera, darkening
of urine, appeared. Objectively: breathing superficial, cardiac tone is rhythmic.
Abdomen is soft, painfull in right hypochondrium, a liver is megascopic, a spleen
A. * Hepatitis A
B. Leptospirosis
C. Cholecystitis
D. Influenza
E. Pseudotuberculosis
242. A patient 42 years old, complaints about dull pain in a right hypochondrium,
weakness, decline of appetite, itching of skin, icterus. Disease began 1 month
ago from protracted àrthralgia, disgust for a meal, strong weakness. An icterus
which now increased considerably appeared three weeks ago. Temperature 36,0
°C. Pulse 56 per 1 min. A tongue is assessed with white cover. A stomach is soft,
sickly in right hypochondrium. A liver +3 cm, spleen is not megascopic. Common
A. Cancer of head of a pancreas
B. * Hepatitis B
C. Hepatitis A
D. Cirrhosis of liver
E. Chronic cholecystitis
243. A patient is 35 years, grumbles about aching pain in right hypocostal area,
nausea, decline of appetite. Beginning of disease binds to appendicitis. After it in
2 months an icterus appeared first. Treated oneself in an infectious department.
In 1 began to notice aching pain in right hypocostal area, in analyses is an
increase of level of bilirubin. Your diagnosis?
A. Calculary cholecystitis
B. Illness of Zhilbera
C. Acute viral hepatitis
D. Chronic cholangitis
E. * Chronic hepatitis
244. The patient T., 35 years, operating trained nurse, appealed to the doctor on the
8th day of gradual development of illness with complaints of a general
weakness, rapid fatigueability, dark color of urine. In the morning noticed the
icterus. On examination temperature of body 36,8 °C. Found out the increase of
A. * liver -+3 sm The changes of what laboratory index most informing at this
AlAT
B. Hemodiastases
C. Protrombin index
D. Cholesterol

E. Alkaline phosphatase
245. A man, 37 years, injection drug addict, practises upon an alcohol, grumbles
about a general weakness, dull pain in right hypocostal area, increase of
stomach, shortness of breath. It is ill chronic hepatitis on an extent 10 years.
Objectively: temperature of body 37,1°C, an icterus is expressed. Liver +6 sm. In
an abdominal region is a free liquid. Positive symptom of fluctuation. What
A. Intensifying of chronic hepatitis C
B. Sharp hepatic insufficiency
C. Gepatokarcinoma
D. Intensifying of chronic hepatitis B
E. * Hepatocirrhosis
246. 23-years old patient during 6 months gets diabetes mellitus in a policlinic the
injections of insulin. A weakness, arthalgia, grew gradually, an appetite
disappeared, then dull pain appeared in an epigastrium after-meal. In 2 weeks
from the beginning of illness noticed the dark color of urine, and afterwards –
icterus on a background which the general state continues to be worsened.
Temperature of body 36,5°C, Ps 58 per 1 min Liver +5 sm, spleen +1 sm What
A. Activity of alkaline phosphatase
B. Activity of lactatdehydrogenase
C. * Activity of ALAT
D. Activity of creatinphosphokinase
E. Activity of amylase
247. 42-years old patient got blood transfusion two months ago. A weakness was
gradually increased, an appetite was worsened, arthalgia, small nausea
appeared. On a 12th day noticed an insignificant icterus, appealed to the doctor.
At examination is the general state fully satisfactory. The temperature of body is
normal, Ps 60 per 1 min, BP 100/70 mm Hg.A liver and spleen is moderate
megascopic, a stomach at palpatory is not sickly. Anti-HCV IGM is found in blood.
A. Neutrocytosis
B. * Leukopenia
C. Lymphopenia
D. Speed-up ESR
E. Aneozinofiliya
248. 25-years old woman during a semiyear got numerous injections concerning
bronchial asthma. An appetite disappeared gradually, a weakness, arthalgia
grew, urine became dark in two weeks, through three – an icterus on a
background which the general state continues to be worsened appeared. The
temperature of body is normal, Ps 62 after 1 mins Moderate sickliness in the
area of pancreas, positive symptom of Voskresenskogo, liver +4 sm, spleen +2
A. * Anti-HBc IGM
B. ANTI-HEV IGM
C. ANTI-CMV IGM
D. HBsAg
E. ANTI-HAV IGM
249. 40-years old patient during 2 days marks absence of appetite, nausea, general
weakness. A year ago carried sharp hepatitis B, avoided a clinical supervision.
Objectively: the temperature of body is enhanceable, skin and sclerotica are
icterus, the megascopic palpatory, to the moderate closeness, sickly liver,
salient from under the edge of costal arc on 3 sm. A spleen is not megascopic.
Wetting moderatory dark colouring. The sickliness of joints is marked at motions.
A. Chronic viral hepatitis, minimum activity

B. Chronic cholecystitis
C. * Chronic viral hepatitis, moderate activity
D. Hepatocirrhosis
E. Chronic viral hepatitis, high activity
250. In 2 months after returning from India, where often drank unboiled water, the
23-years-old pregnant nauseated, strong general weakness, head pain, later the
temperature of body rose to 38,6°, which stuck to within a week. An icterus
appeared on a 6th day, the general state continued to be worsened. On the 12th
day of illness the general state heavy. Euphoria. Vomiting at night. Complete
fastidium. Bright icterus, signs of hemorragic syndrome, tachycardia. BP 110/60
mm Hg, temperature of body of 37,8°C. A liver is insignificantly megascopic,
soft, painfull, spleen +2 sm There is neutrophilic leykocytosis in the global
A. * Hepatitis A
B. Malignant icterus of pregnant
C. Typhoid
D. Mechanical icterus
E. Leptospirosis
251. A patient is 25 years, appealed to the internist with complaints about a general
weakness, worsening of appetite, feeling of weight in right hypocostal area.
Treated oneself on an occasion ARVI ambulatory 3 days. A feel was worsened,
nausea, pain in right hypocostal arrea, ochrodermia of sclerotica, darkening of
urine, appeared. Objectively: breathing normal, cardiac tones are rhythmic. A
stomach is soft, sickly in right hypocostal area, a liver is megascopic, a spleen is
A. * Hepatitis A
B. Leptospirosis
C. Calculary cholecystitis
D. Flu
E. Pseudotuberculosis
252. ?What is the most credible treatment of a patient with complaints about general
weakness, fatigueability, somnolence, pain in chest,during 2 weeks has anxiety,
fear, depression and disorders of memory and aphasia. Untidiness appeared 5
days ago.
A. Acylovir
B. * Zidovudin, didanosin
C. Cerebroprotector
D. Neuroleptic
E. Antidepressants
253. What is the most reliable diagnosis for Patient with complaines of fever, night
sweating, a sharp decrease in body weight, pain in muscles, throat, joints,
photophobia, recurrent diarrhea. When inspection revealed a generalized
A. * lymphadenopathy.
AIDS What is the most reliable diagnosis.
B. Lymphogranulomatos
C. The tumor of the digestive system
D. Tuberculosis
E. Salmonellosis
254. What is character for I clinical stage of AIDS, in obedience to clinical
classification of stages of HIV-infection for adults and teenagers, which is
A. * developed
Loss of mass byof
the experts
body of WHO
5 % from (in 2002)?
initial
B. Purulent defect of skin and mucus (seborrhic dermatitis, mycotic defect of nails)
C. An episode of herpes zoster during the last five years
D. Risiding of infection of upper respiratory tracts (for example, bacterial sinuitis)

E. Minimum defeats of mucus (recurrent ulcers of mucus shell of oral cavity)


255. The sick grumbles about the prolonged cough, more than half-year, rising
temperature of body to 38 °C, enlargement of peripheral lymphnode, frequent
herpetic wide-spread pouring out with considerable lowering of body mass. In
³mmunogram correlation of T-helper to T-suppression is 0,3. It takes place
A. becausecells
Infects infestant:
with the receptors of CD22
B. Induces proliferation of Ò-helpers
C. Induces proliferation of T-suppressors
D. Infects cells from receptor CD8
E. * Infects cells from receptor CD4
256. On a background of prolonged treatment for HIV patient, appeared ulcer on the
mucous cell of mouth. At an objective examination doctor have found out
erosions on the mucous, hyperemic and filling out mucus cell of oral cavity,
tongue without raid, with a smooth surface. About what complication is possible
A. to think?
Leptospirosis
B. Acute herpes
C. Stevens-Johnson syndrome
D. * Candidos stomatitis
E. Layel syndrome
257. Immediately after a contact with blood and other biological liquids it is
necessary to wash the muddy areas of skin with water and soap and to begin a
postcontact prophylaxis, antiretroviral preparations not later than
A. * 24-36 hrs
B. 36-48 hrs
C. 48-60 hrs
D. 60-72 hrs
E. 72-86 hrs
258. A youth with the catarrhal phenomena found out enlargement of axillary and
submandibular lymphatic node, hyperplasia of oral cavity with the magnificent
raid of gum-blush on them as points and spots, presence of mucus cell pouring
out, increased liver and spleen size.What additional researches must be
A. appointed
A test to HIVto do for diagnosis?
B. IFA for the exposure of antibodies toHIV
C. IFA for the exposure of antibodies to the virus of Epstien-Bar
D. Analysis in the presence of mononuclear antibodies
E. * All the above
259. A sick entered permanent establishment with complaints about general
weakness, increase of temperature, pain in throat. Objectively: the mucus cell of
retropharynx is bright red, on oral cavity are raids of gum-blush, taken off easily,
discovered enlargement of all groups of lymphnode, 2-3cm in a diameter, dense,
elastic little painful, not soldered between itself. Liver is enlarged on 3cm, spleen
A. * Infectious mononucleosis
B. Diphtheria
C. Acute leukosis
D. Quinsies
E. Adenovirus infection
260. Combinations of loss of body mass, sweating, recurrent àpthus stomatitis and
girdle herpes and lymphadenopathy, allowed a physician to assume HIV-
infection. It can be discovered at an additional inspection:
A. Increased correlation of CD4/CD8 lymphocyte

B. * Diminished correlation of CD4/CD8 lymphocyte


C. Change of neutrophil formula to the left
D. Increased correlation of T8/T4 lymphocyte
E. Diminished correlation of T8/T4 of lymphocyte
261. Sick 28 years, prostitute of 2 years admit on neareast hospital with complaints
of weakness, coughing, producing serosal sputum, mass in breasts, short
breathing, chills at night, fever, pain in joints, rise of white color on mucus
cavities of mouth. For 5 months the sick became thin on 12 kg, with appetite
diminished, it become hard to swallow hard food. At examination: on the soft
palate to the back wall| of glottis rise of white color and lymphadenopathy.
Above the lungs, from the corner of shoulder-blade, dulling percutory sound.
A. Visceral leshmaniasis
B. * HIV infection. Pneumocystis pneumonia. Candidiasis of glottis
C. HIV infection. Pneumococal pneumonia. Candidiasis of glottis
D. Megakaryoblastoma
E. HIV infection. Pneumococal pneumonia. Gonococcal arthritis. Candidiasis of
262. glottis
A woman M., 24, appealed to the doctor in connection with the protracted fever,
sweating at night. For the last three months became thin on 7 kg. On objective
research the increase of all groups of lymphatic nodes, hepatospleen syndrome
is found. In blood: Leu – 2,2x109, anaemia. What disease must be suspected?
A. * HIV infection
B. Megakaryoblastoma
C. Tuberculosis
D. Infectious mononucleosis
E. Sepsis
263. Student L., 20 years old, during a month disturb with considerable fatigue
ability, fever with 39 °C, profuse sweating, lethargy. Objectively: all groups of
peripheral lymphatic node are enlarged, Candidiasis of oral cavity, herpetic
pouring out are megascopic on lips. In the global analysis of blood: leucocytes of
3,3x109 g/L, ESR 15 mm/hr, B-lymphocyte 12 %, correlation of CD4/CD8 0.8.
A. Megakaryoblastoma
B. Malaria
C. * HIV/AIDS
D. Urogenital chlamydiasis
E. Acute brucellosis|
264. A patient, 28 years old, grumbles about the enlargement of lymphnodes during
half-year, making progressive weakness, sweating, periodic fever upto 38 °C.
Decreased feed, seborhic dermatitis is wide-spread, the enlargement of neck,
axillary and inguinal lymph nodes with diameter 2-2.5cm and unpainful. What
A. research
Biopsy of is necessary for diagnosis?
lymphnode
B. Hemanalysis on sterility
C. * nalysis in the presence of antibodies to HIV
D. Sternal puncture
E. Analysis in the presence of antibodies to the Epstein-Bar virus
265. For a man with 30 years, it was half-year ago discovered positive reaction on
HIV. Last 3 months complaints about general weakness, fatigueability,
somnolence, pain of chest. Last 2 weeks with anxiety, fear, depression, disorders
of memory and aphasia, untidiness appeared 5 days ago. What does it need to
A. take for treatment?
Acylovir
B. Cerebroprotector

C. Antidepressants
D. Neuroleptic
E. * Zidovudin, didanosin
266. Patient P., 21 year,complaint about diarhhea that lasts for one and half months,
changes sometimes, on emptying there are admixtures of blood and mucous,
loss of body mass with 13 kg, weakness, subfebrile temperature of body,
recurrent herpes. It is discovered generalised lymphadenopathy, increase of liver
size on 2 cm. Blood test: Er 4,4.1012 g/l, Hb 115 g/l, ESR – 15 mm/hr, L 10,0.109
A. Shigellosis
B. * AIDS
C. Infectious mononucleosis
D. Salmonellosis
E. Ameobiasis
267. Sick O., 25 years, during a month disturb with considerable fatigue ability, fever
up to 39 °C profuse sweating, lethargy. Objectively: all groups of peripheral
lymphatic node are enlarged, candidiasis of oral cavity, herpetic pouring out
with megascopic on lips. In the global analysis of blood: leucocytes 3,3?109 g/l,
A. ESR – 15m/hr,
Herpetic B-lymphocyte 12 %, a decline of CD4 to 600/mm. Most credible
infection
B. Malaria
C. Urogenital chlamydia
D. * HIV/AIDS
E. Acute brucellosis
268. HIV positive patient, 28 years, in the past with IV drug addiction, grumbles about
short breathing, unproductive cough, fever with 37,5 °C during 2 months.
Objectively: skin is pale, in lung the hyposthenic breathing, especially in lower
lobe, short breathing with 24 /min. Roentgenologic chromophilic bilateral
A. infiltrations aspreparations.
Antibacterial “wings of butterfly”.
InhibitorsPick up the most effective treatment:
of transcriptase
B. Inhibitors of proteases. Inhibitors of transcriptase
C. Transplantation of marrow. Inhibitors of transcriptase
D. Antiviral gamut-globulin. Vitamins of group A, C. Inhibitors of proteases
E. * Inhibitors of proteases. Inhibitors of transcriptase. Antibacterial drugs
269. For a patient 20 year, fever during 5 days, pharyngalgia at swallowing, pains in
muscles. At examination, hyperemic mucus with festerings stratifications which
are easily taken off, neck, axillary and cervical lymphnode are enlarged, liver is
enlarged by 1,5cm, the edge of spleen is palpable. What additional researches
A. must be appointed?
Puncture of lymphatic node with a subsequent microscopy
B. Ultrasound of organs of abdominal region
C. * A hemanalysis on mononuclear antibodies
D. Occupied blood
E. RPGA with the viruses of flu
270. Student L, 20 years, during a month disturb with considerable fatigue, feverence
body temperature with 39 degree Celsus, profuse sweating lethargy.Objectively:
all groups of peripheral lymphatic node are enlarged, candidiasis of oral cavity,
herpetic pouring out with megascopic on lips. In the global analysis of blood:
A. leucocytes of 3,3?109 g/l, ESR 15 mm/hr, B-lymp 12 %. Most credible diagnosis
Megakaryoblastoma
B. Malaria
C. * HIV/AIDS
D. Urogenital chlamydiasis
E. Brucellosis

271. Patient L,18 year, entered permanent establishment with complaints about pain
in chest, general weakness, increase of temperature to 38 °C, during 6 days
present pharyngalgia. Objectively:all groups of lymph node are enlarged with 1-3
cm in diameter, elastic, little painful, unsoldered between itself. Liver is enlarged
on 3cm, spleen – on 1cm. In blood: leucocytosis, mononuclear antibodies – 15 %.
A. Diphtheria
B. Adenovirus infection
C. Quinsy
D. * Infectious mononucleosis
E. Acute lympholeukosis
272. Patient A., 28 year, grumbles about the increase of lymphnode on a neck during
half-year, making progress weakness, sweating and periodic getting up of
temperature of body to 38 °C. Loss of weight, seborrhic dermatitis is wide-
spread, the enlarged painless neck, axillary and sub mandibular lymph node by
A. 2-2,5cm.in
Analysis What research is
the presence ofnecessary
antibodiestotoappoint?
CMV
B. A biopsy of lymphatic node for the exposure of cells of varicella-zooster
C. Analysis in the presence of antibodies to HIV
D. * Analysis in the presence of antibodies to HIV
E. Analysis in the presence of antibodies to Epstein-Barr virus
273. Patient 30 years, delivered in an infectious separation with ambulance in a grave
condition. At examination: temperature of body 38,9 °C a patient is exhausted,
put on the bed, skin covers pale, in the area of peripheral veins are tracks of
injections. In an oral cavity are plenty of carious teeth, many raids on mucus
cell.Determined enlarged submandibular, supraclavicular and axillary lymph
nodes, painless not fixed with subject. In lung, breathing is vesicular, breathing
A. Megakaryoblastoma
B. Herpetic encephalitis
C. * AIDS
D. Syphillis
E. Lympholeucosis
274. Student of 22 year, during a month disturb with considerable fatigue ability,
fervescence of body 39 °C, profuse sweating, loss of weight.Objectively, all
groups of peripheral lymphatic node are enlarged, candidiasis of oral cavity,
herpetic pouring out are megascopic on lips.In the global analysis of blood:
leucocytes 3,3.109 g/l, ESR 15mm/hr, B-lympho 12 %, correlation of CD4/CD8
A. Megakaryoblastoma
B. Malaria
C. * HIV/AIDS
D. Urogenital chlamydia
E. Acute brucellosis
275. Patient O, 18 years, over 4 months grumble about obtructive cough, fervescence
to 38 °C, treatment concerning interstitial pneumonia, did not give an effect.
There were a few episodes of the wide-spread herpetic pouring out, mass of
A. body reduced.
Increase Research
of number of immunogram more reliable finding
of CD4-lymphocyte
B. Increase of immunoregulatory index of CD4/ CD8
C. * Decrease of number of CD4-lymphocyte
D. Decrease of number of CD8-lymphocyte
E. Increase of indexes of hypersensitiveness of slow type

276. For a man 30 years, half-year ago there was a positive reaction on HIV. Last 3
months complaints about general weakness, somnolence, pain of chest, during
2 weeks developed anxiety, fear and depression. Disorders of memory and
A. aphasia, untidiness
Dementsia at AIDS appeared 5 days ago.To set a diagnosis.
B. Somatoform depression
C. Organic psychic syndrome
D. Anxiously depressed syndrome of HIV infection
E. * Patient has encephalopathy (AIDS-related complex)
277. For a patient I., 37 years, during 3 months deceleration of muscular and motive
reactions, names, addresses and cognitive memory functions were violated. A
constraint, somnolence appeared and untidiness developed, indifferent attitude
toward circumferential state. Loss of weight 12 kg. At examination generalised
A. lymyphadenopathy. Put a clinical diagnosis:
Tuberculosis
B. Herpetic encephalitis
C. * AIDS
D. Syphillis
E. Lympholeucosis
278. Student O, 37 years during two years disturb with considerable fatigue, frequent
fever to 39 °C profuse sweating, loss of weight. Objectively, the lymphatic node
of neck and sub mandibular are enlarged, candidiasis of oral cavity, herpetic
pouring out on lips. In the global analysis of blood: leucocytes 3,3x109 g/l, ESR –
A. 15mm/hr, B-lympho 12 %, correlation.Most credible diagnosis?
Megakaryoblastoma
B. Malaria
C. * HIV/AIDS
D. Urogenital chlamydia
E. Sharp brucellosis
279. A sick of 18 years, grumbles about an obtructive cough for 6 months, fever to 38
°C, enlargment of lymphatic nodes, frequent herpetic wide-spread pouring out,
considerable lowering of mass of body. In immunogram correlation of CD4 to
CD8infestant
A. * An is 0.5. Itinfects
takes place because
cells with of
the receptors of CD4
B. Induces proliferation of Ò-helper
C. Induces proliferation of T-suppressor-cell
D. Infects macrophagal cell
E. Stimulates the synthesis of leukotriene
280. Student I, 25 years, during a month disturb rapid increasing of body temperature
to 39 °C profuse sweating, loss of appetite. Objectively, all groups of peripheral
lymphatic nodes are enlarged, candidiasis of oral cavity, herpetic pouring out
are megascopic on lips.In the global analysis of blood, leucocytes- 3.3x109 g/l,
A. ESR 15mm/hr, B-lymphocyte 10 %.Most credible diagnosis
Megakaryoblastoma
B. * HIV/AIDS
C. Cytomegaloviral infection
D. Urogenital chlamydia
E. Acute brucellosis
281. A patient E, 40 year, appeared on hospital with complaints about fever, sweating
at night, loss of wieght, pain in muscles,throat and joints and recurrent diarhoea.
At examination found out generalised lymphadenopathy. What is most reliable
A. diagnosis?
Salmonellosis
B. Lymphogranulomatosis
C. * AIDS

D. Tumour of gastrointestinal tract


E. Amebiasis
282. Sick, 16 year, hospitalized on the 5th days of illness with complaints about
moderate pain of head in frontal-temporal area, book-mark of nose pharyngalgia
at swallowing, pain in the area of the left eye, fervescence to 38.1-38.5°C. The
state at entrance is satisfactory.The nasal breathing is laboured and abundant
mucus excretions from a nose, hyperemia of mucus shell on posterio pharyx,
A. Flu
B. Infectious mononucleosis
C. Enterovirus infection
D. * Adenovirus infection
E. HIV infection and acute retrovirus syndrome
283. Patient A, 20 year appeared in hospital with complaints about moderate
pharyngalgia, pain in chest, general weakness, fervescence of 38,9 °C. Became
sick three days ago. Objectively, discovered enlarged axillary, sub-mandibular||
and cervical lymphatic node.Hepatospleenomegaly sclerotic and skin changes,
in the throat characteristic for quinsy.At investigation of blood,
A. * Infectious mononucleosis
B. Follicular tonsillitis
C. Adenoviral infection
D. HIV and acute retrovirus syndrome
E. Viral hepatitis
284. Patient 17 years, has pharyngalgia, temperature of body rise to 38 °C.
Objectively hyperemia of the mucus cell of oral cavity, all groups of lymphatic
nodes are enlarged to 1- 3cm in diameter, not soldered between itself, little
pianful, and elastic. Liver is enlarged on 3 cm and spleen +1cm. What additional
A. research must
Examination forbemycosis
appointed above all the things?
B. * IFA for the exposure of antibodies to HIV
C. Puncture of lymphatic node with subsequent microscopy
D. A hemanalysis on anti mononuclear antibodies
E. RPGA with the viruses of flu
285. A patient concerned about attacks of fever, which are repeated every third day.
There are jaundice of sclera and skin, hepatosplenomegaly. Which of the
A. following
Sepsis diagnosis is most likely?
B. * Malaria
C. viral hepatitis
D. Hemolytic anemia
E. Leptospirosis
286. ?At a patient with a pediculosis the temperature of body rose to 41,6 °C,
appeared headache, euphoria, in 4th days from the beginning of illness –
rozeola-petechial rash. Reaction with rhycetsia of Provachec 1:640, IGG – 89 %.
A. What is your diagnosis?
Flu
B. Typhoid fever
C. Meningococcal infection
D. * Epidemic typhus
E. Leptospirosis
287. A patient 42 years old, for a week every 48 hours appear attacks chills, which
are replaced by heat. Body temperature rises to 40 °C, in 3-4 hours was a severe
sweat. Worsened appetite, a common weakness. Skin pale.
Hepatosplenomegaly. What is the treatments tactics?

A. * Nomination delagila
B. Introduction immunoglobulin
C. Appointment of antibiotics
D. Detoxification
E. Introduction glucocorticoisteroids
288. Patient A., 37 years old, entered to infectious hospital on the third day of disease
in the severe condition. He complaints about the high fever with chills and
sweat, general weakness, pain in right under a rib. Objectively: temperature of
body 41 °Ñ, icterus of skin, liver +2 cm, pain at palpation in abdomen, positive
symptoms of Ortner and Mussy, a spleen is normal, tachycardia. What is the
A. Malaria
B. * Cholangitis
C. Viral hepatitis
D. Sepsis
E. Leptospirosis
289. A patient L., 32 years old, who complains of severe headache and fever, on the
6-day of illness positive agglutination test with rickettsia reaction. What is the
A. Vector of the disease:
Flea
B. Fly
C. Mosquitoes
D. Bee
E. * Lice
290. Patient D., 39 years old, entered to infectious hospital on the second week days
of disease in the severe condition. Local habitant, nowhere arrived. She
complaints for the high fever with chills and sweat, general weakness. Attacks of
fever without correct periodicity. Objectively: the temperature of body – 41 °Ñ,
subicterus of sclera, tachycardia, Pasrernacky‘s symptom positive, liver + 2 cm,
A. Malaria
B. Cholangitis
C. Pyelonephritis
D. * Sepsis
E. Leptospirosis
291. A patient 29 years, a few days ago, a chill, the temperature for 2-3 days
increased to 39-40 ° C. There is hyperemia and edema, significant sclera like
"drunk" person and "rabbit" eyes. On the third day of illness – on mucous of soft
palate, parenthesis bright red enantema is seen. At 3-4-day patient when trying
to protrude the tongue, there was hypermovement, tremor, rejecting it to the
A. Air-dropping
B. Fecal-oral
C. Contact Residential
D. Parenteral
E. * Vector borne
292. A patient 29 years, a few days ago, a chill, the temperature for 2-3 days
increased to 39-40 °C. There is hyperemia and edema ,significant sclera like
"drunk" person and "rabbit" eyes. On the third day of illness – on mucuos of soft
palate, parenthesis bright red enantema is seen. At 3-4-day patient when trying
to protrude the tongue, there was hypermovement, tremor, rejecting it to the
A. * Epidemic typhus
B. Typhoid fever
C. The disease Brill

D. Paratyphoid A
E. Paratyphoid B
293. A patient at the 4-day fever has profuse rosy-petechia exanthema predominantly
located on the lateral surface of the torso and limbs flexion surfaces. Hyperemic,
vascular conjunctivitis, petechiae transition of anterior fold conjunctiva. What
A. kind of illness
Typhoid fever can you think?
B. * Typhus
C. Measles
D. Haemorrhagic fever with renal syndrome
E. Crimean hemorrhagic fever
294. Patient B., complains of headaches, delirium. When inspection: a body
temperature of 39 ?C, initiated, a person hyperemic, positive symptom
Govorova-Godele, petechia rash on the trunk, limbs, tachycardia, hypotension,
hepatosplenomegaly. Drunker, drug user, living in the basement room type.
A. What is your preliminary diagnosis?
AIDS infection
B. Influenza
C. Leptospirosis
D. * Epidemiys Typhus
E. Alcohol psychosis
295. A patient 28 years old, of no fixed abode, hospitalized with a preliminary
diagnosis «flu», a 5-day illness appeared rosy-petechia rashes on the body and
interior surfaces of the extremities. The temperature of 41 °C, euphoria,
hyperemia person, redness sclera, tongue tremor, tachycardia, enlarged spleen;
A. excitement.
Typhoid feverWhat is the likely diagnosis?
B. Leptospirosis
C. Alcohol deliry
D. Measles
E. * Epidemic typhus
296. A patient 28 years at the 4-day fever has profuse rosy-petechia exanthema
predominantly located on the lateral surface of the torso and limbs flexion
surfaces. An individual patient hyperemic expressed vessels conjunctival
injection, petehies transition to fold conjunctiva. Analysis of urine protein single
A. hyaline and
Typhoid granular cylinders. What kind of illness can think?
fever
B. Haemorrhagic fever with renal syndrome
C. Crimean hemorrhagic fever
D. Measles
E. * Epidemic typhus
297. The patient, a train conductor, 39 years old, hospitalized for a 4-day illness with
complaints of headache, weakness, dizziness, sweating, insomnia, fever.
Hyperemic, edema, conjunctivitis. At the transition fold conjunctiva – single
petehies. At the skin torso, chest, abdomen, limbs abundance rosy-petehia rash.
Tachycardia. AD 100 and 60. Tremor of the tough. Palpable liver, spleen. Stool
A. Leptospirosis
B. Typhoid fever
C. Influenza
D. Meningoccemia
E. * Epidemic typhus

298. A patient 30 years on 9th day of illness that began gradually, the slow rise of
fever and intoxication, a painless rosy rashes on the skin of the abdomen.
OBJECTIVE: pale, temperature 40 ° C, pulse 80/min, AD 100 and 65 mm RT. Art.
Language is coached, stomach bubbles, enlarged spleen and liver. For what
A. * disease you
Typhoid fevercan suggest in the first place?
B. Epidemic typhus
C. Measles
D. Scarlet fever
E. Sepsis
299. Patient 20 years, complained about the high temperatures of up to 39 ° C,
headache in the frontal area, pain in the eyeball, photophobia, pain in muscles,
dry cough. Acutely ill the day before. Objective: condition serious, hyperaemic,
eyes shining, vascular injection sclera. Pulse 96/min, rhythmical. Tone heart
weakened. In the lungs scattered dry bubbling rale. Faces hyperemic, granular.
Meningeal symptoms are not present. Blood tests: Leu. 3*109, eoz. 1 %, band
A. Measles
B. * Influenza
C. Meningococcal disease
D. Pneumonia
E. Typhus
300. atient K., 23 years old., hospitalized on the 3rd day of illness, which was
accompanied by mild running nose, high fever to 40,2 °C, headache and
hemorrhagic rash on the skin. In the 2 hours after the introduction of penicillin
blood pressure dropped to 40 and 10 mm RT. Art. Distal pulse and meningeal
A. * Meningoccocemia,
signs are not defined. What is the diagnosis
infectious-toxic shock in a patient?
B. Epidemic Typhus, severe course
C. Measles, severe course
D. Scarlet fever, severe course
E. Flu, anaphylactic shock
301. Male 45 years old, fell ill after 2 weeks after returning from Afghanistan, where
six months ago underwent malaria. In return were found lice. Suddenly, there
were severe headache, weakness, body temperature 39 ?C with a temporary
decrease in the 4th day of illness, followed by general weakness, much
intoxication, headache, appeared on the body abundance polymorphic rash. At
the 7-day state of heavy, the phenomenon of encephalitis, excitation,
hallucinations periodically. Objective – erythematos-petechial exanthema. The
temperature of the body 40,1 0C, pulse was 136 per 1 min, blood pressure
A. At typhoid fever
B. At the three-day malaria
C. At tropical malaria
D. * At epidemic typhus
E. At brucellosis
302. Patient P., 76 years old, complained during 7 days for the permanent increase of
temperature to 38,2-38,7 °C, headache, insomnia, horrific dreams, dry cough,
myalgias, artalgias. Polymorphic rash appeared on a trunk on the 4th day of
diseases. In childhood was ill by epidemic fever, three-day malaria. Temperature
of body – 38,4 °C, pulse – 98 per a min, hepatolienal syndrome, bilateral
pneumonia confirmed roentgenologic. In the analysis of blood moderate
A. * Brill‘s disease
B. Typhoid fever
C. Lime‘s disease

D. Malaria
E. Leptospirosis
303. Patients D., 30 years old, became ill saddenly, when the temperature of body
rise to 40,2 °C, appeared headache, weakness, euphoria, injection of vascular
sclera, hyperemia of face, appears the positive symptom of Govorova-Godele.
Reaction with Rickettsia Provachecky 1:160, IGG – 87 %. What is your
A. diagnosis?
Brill‘s disease
B. Typhoid fever
C. Paratyphoid A
D. Paratyphoid B
E. * Epidemic typhus
304. atient K., 23 years old., hospitalized on the 3rd day of illness, which was
accompanied by mild running nose, high fever to 40,2 °C, headache and
hemorrhagic rash on the skin. In the 2 hours after the introduction of penicillin
blood pressure dropped to 40 and 10 mm Hg. Distal pulse and meningeal signs
are not defined. What
A. * Meningoccocemia, is the diagnosis
infectious-toxic in a patient?
shock
B. Epidemic Typhus, severe course
C. Measles, severe course
D. Scarlet fever, severe course
E. Flu, anaphylactic shock
305. Patients D., 30 years old, became ill saddenly, when the temperature of body
rose to 40,2 °C, appeared headache, weakness, euphoria, injection of vascular
sclera, hyperemia of face, appears the positive symptom of Govorova-Godele.
Reaction with Rickettsia Provachecky 1:160, IGG – 87 %. What is your
A. diagnosis?
Brill‘s disease
B. Typhoid fever
C. Paratyphoid A
D. Paratyphoid B
E. * Epidemic typhus
306. Patient G., 28 years old, sailor, has been ill in 18-day illness, complaining of high
fever, headache, weakness. Disease began as a chills during an hour, then the
temperature had risen to 39,8 ?C. In next 5-6 days, chills repeated every day in
the middle of the day, after which the temperature rose to 40-41 °C, such
attacks fever occur every other day. Face is hyperemic, herpes, vascular
injection of sclera. Tons of the heart muted, rhythmic, pulse 140 for 1 min, AD
A. * Primaquine
B. Fansidar
C. Metakelfin
D. Quinine
E. Tetracycline
307. A citizen of Nigeria, 19 years old, came to study in Ukraine. After 10 days of
appeared a strong chill, which lasted about 1 hour and changed the feeling of
heat. There has been a headache, slice, nagging pain in the muscles. Attack of
such clinical symptoms rapid again after 2 days. OBJECTIVE: body temperature
39,6 °C, sclera subicteric, appeared lips herpes. Spleen significantly increased,
A. * Microscopy of peripheral blood
B. Microscopy of centrifuge of urine
C. Lumbar puncture
D. Observation of oculi
E. USD abdomen

308. atient G., 20 years old, six months ago returned from Africa. A month later he
introduced the three-day clinical symptoms of malaria. Whether is it necessary
to carry out epidemic contacts that the patient had?
A. You do not need because there is no vector
B. You need because there is vector
C. * You do not need because reconvalence not cause
D. It should be, because rekonvalence allocates cause
E. You do not need because epidemic activities inefficient
309. A patient G., 42 years old, for a week every 48 hours had attacks of chills, which
is followed by heat. Body temperature rises to 40 °C, in 3-4 hours was a severe
sweat. Worsened appetite, a common weakness. Skin pale.
Hepatosplenomegaly. What are the most effective method for verification of
diagnosis. and a thick smear of blood drops
A. * Microscopy
B. Microscopy hanging drops
C. An overall analysis of blood
D. Bacteriological method
E. Enzyme multiplied immunoassay
310. Patient A., 27 years old, was admitted to the isolation hospital for the second
week of the beginning of the disease. Complaints of fever to 38 0C with chills,
general weakness, nausea, vomiting. The temperature lasted 3 hours, then
noticed darkening of urine, and a day yellowed. OBJECTIVE: Normal body
temperature, icterich sclera and skin, bradycardia, liver +2 cm, spleen palpable
A. * The definition of markers of viral hepatitis
B. Parasitoscopy of blood
C. An overall analysis of urine
D. Bacteriology of stool
E. Biochemical blood test
311. A patient was disturbed by attacks of fevers, repeated periodically every third
day. The icterus of sclera and skins, is observed, hepatosplenomegaly, is found
A. * on palpation. What is the diagnosis?
Malaria
B. Sepsis
C. Viral hepatitis
D. Gemolitic anaemia
E. Leptospirosis
312. ?A patient is disturbed by attacks of fever. The icterus of sclera and skins is
observed, hepatosplenomegaly is found on palpation. What is possible diagnosis
A. accept?
Leishmaniosis
B. Leptospirosis
C. * Sepsis
D. Malaria
E. Viral hepatitis
313. A patient sharply had a chill, head pain, vomits, temperature of body rose up to
38,5 °C. Till evening rigidity of muscles, Kernig’s symptom appeared. Herpetic
blisters are marked on mucous of lips and nose. Neurological symptoms is not
A. found outencephalitis
Herpetic expressed. What disease will you suspect?
B. Abscess of brain
C. * Meningoencephalitis
D. Hemorrhage in a brain
E. Subarachnoid hemorrhage

314. Examination is delivered in a clinic. Objectively: temperature 40,2 °C. Expressed


meningeal symptoms. Light, haptic, pain hypesthesia. Blood: leucocytes – 5
thousand/l, ERS-19 mm/hr. A neurolymph under high pressure, turbid,
lymphocytosis, some increasing of protein, chlorides. What is your diagnosis?
A. Meningococcal meningitis
B. * Viral meningitis
C. Tubercular meningitis
D. Staphylococcus meningitis
E. Pneumoccocal meningitis
315. A 17 years old boy, after tick bite got ill suddenly: the temperature rise up-to
40,3 °Ñ, extremely sharp headache, makes him to yell. The patient is agitated
and vomits frequently. Tremor of fingers of extremities is observed. Meningeal
A. * signs are positive. What is the most credible diagnosis?
Viral meningoencephalitis
B. Leptospirosis
C. Measles
D. Meningococcal infection
E. Flu with a hemorrhagic syndrome
316. A citizen of Nigeria, 19 years old, came to study in Ukraine. After 10 days a
strong chill appeared. There has been a headache, slice, nagging pain in the
muscles. OBJECTIVE: body temperature 39,6 °C, sclera subicteric, herpes of lips.
Symptoms of bronchitis were present. Spleen is significantly increased, dense,
A. the liver
USD is normal. What investigation is necessary to do?
of abdomen
B. Lumbar puncture
C. * Microscopy of tissue slads
D. Microscopy of peripheral blood
E. Observation of e.coli
317. A patient became ill sharply. Severe pain of head, frequent vomits appeared in
the morning. Temperature of body rise up to 39,9 °C. Adopted febrifuge,
however much the state got worse. Till the evening patient lost consciousness.
Excited, sharply expressed meningeal signs. What is most reliable diagnosis?
A. * Viral meningoencephalitis
B. Meningococcal infection, meningitis
C. Status typhosus in epidemic typhus
D. Sepsis, infectious-toxic shock
E. Status typhosus in typhoid fever
318. A patient became sick 2 days ago suddenly. Temperature of body rise up to
41°Ñ, headache, vomit, positive Kernig’s and Brudzinsky’s symptoms appeared.
In cerebrospinal liquid: cytosis 7 000 in 1 mkl, 90 % are lymphocytes, some
A. increasing of protein, sugar, chlorides. What is clinical diagnosis?
Endocarditis
B. * Meningeal form of viral encephalitis
C. Infectious-toxic shock
D. Meningococcemia
E. Meningism
319. A patient C., 25 years old, fell suddenly ill. Every morning severe headache,
frequent vomiting, temperature of the body is 39,9 °C. Adopted fatigue, then
state got much worse. In the evening lost of consciousness. Expressed muscles
pains of back and head. Positive Êårning’s symptom. Leukocytes – 8,0?109.
A. What is the
Epidemic most reliable
typhus, diagnosis?
typhus state
B. Flue
C. * Viral menigoencephalitis

D. Bacterial menigoencephalitis
E. Sepsis, infectious-toxic shock
320. A patient W., 40 years old, stay sick during few weeks: insignificant weakness,
body temperature – 38 °C, at right forearm – 2 ulcers of diameter 1,5 cm,
covered by scab with pus under it, surrounded by infiltration. What is the most
A. credible diagnosis?
Eryzipelas
B. Tularemia, ulcerous-bubonic form
C. Skin leishmaniosis
D. Staphylococcal carbuncle
E. * Anthrax
321. A patient became sick 2 days ago suddenly. Temperature of body rise up to 41
°Ñ, headache, vomit, positive Kernig’s and Brudzinsky’s symptoms appeared. In
cerebrospinal liquid: cytosis 7 000 in 1 mkl, 90 % are lymphocytes, some
A. increasing of protein, sugar, chlorides. What is clinical diagnosis?
Endocarditis
B. Meningococcemia
C. Infectious-toxic shock
D. * Meningeal form of viral encephalitis
E. Meningism
322. A patient C., 25 years old, fall suddenly ill. Every morning severe headache,
frequent vomiting, temperature of the body is 39,9 °C. Adopted fatigue, then
state got much worse. In the evening lost of consciousness. Expressed muscles
pains of back and head. Positive Êårning’s symptom. Leukocytes – 8,0?109.
A. What is the
Epidemic most reliable
typhus, diagnosis?
typhus state
B. Bacterial menigoencephalitis
C. Flue
D. Sepsis, infectious-toxic shock
E. * Viral menigoencephalitis
323. At a patient with meningoencephalitis 44 years old, rise up general clonic-tonic
cramps, abundant sweat, hyperemia of the face, bradycardia quickly changed on
tachycardia, violation of breathing (Cheyne-Stokes type). What complication
A. develop?
welling and edema of cerebrum
B. * Wedging of cerebrum in the cervical channel
C. Hypovolemic shock
D. Waterhause-Friedrichsen syndrome
E. Infectious toxic shock
324. At a patient, 35 years, after the trip to Japan acute beginning of disease
happened. Body temperature – 39,5 °Ñ, headache, which is accompanied with
vomits, myalgia. Hemorrhagic rash, that are localized on sclera. Rigidity of
cervical muscles, Kernig and Brudzinsky symptoms are expressed. Spinal liquid
A. without any changes.
Meningococcal infectionWhat is most credible diagnosis?
B. Leptospirosis
C. Epidemic typhus
D. Typhoid fever
E. * Viral meningoencephalitis
325. In a patient, 27 y.o., after tick biting, headache which accompanied by nausea,
repeated vomits, hyperesthesia, photophobia appear. At a review: feet are
bended to the trunk, expressed rigidity of muscles of the back of head, positive
symptoms of Kerning’s, Brodzinsky’s. It is not found out paresises. Select a basic
A. neurological
Root syndromesyndrome.
B. * Meningeal syndrome
C. Syndrome of liquor hypertension
D. Vegetative crisis
E. Syndrome of liquor hypotension
326. Patient A., 43 years old, was admitted to infectious hospital with complaints on a
subfebrile temperature and headache. Fill bad some months. By examination
observe ulser on forearm with infiltration around it and painful. Regional
lymphadenitis is not present. Hepatolienal syndrome. It is known from
A. anamnesis,
Cancer that a patient visited India a year ago. About what disease is it
of skin
B. Erizipeloid
C. * Skin leishmaniosis
D. Eryzipelas
E. Anthrax
327. Patient A., indian student, complaints about redness of skin and edema at the
right cheek. During examination: body temperature – 38,7 ?C; gray-red
tuberculum, covered with peels. Your preliminary diagnosis?
A. Herpetic infection
B. Anthrax, skin form
C. * Skin leishmaniosis
D. Eryzipelas, hemorrhagic form
E. Phlegmona
328. Patient B., 38 years, delivered by emergency doctor, anamnesis is unknown.
However, according to neighbours, patient 2 days ago was healthy. At a review:
consciousness is absent, motion of left overhead and lower extremities is
absent, increasing of muscular tone is marked in the same extremities.
Periodically are clonic-tonic cramps in right extremities. Expressed asymmetry of
person. Sharply expressed rigidity of muscles of the back of neck, positive
Êårnig symptom, positive overhead and lower Brudzinsky symptom. Acrocyanîsis
is marked, temperature of body 38,8 °Ñ. Breathning 36/min, unrhythmical.
Vesicular breathing during auscultation. Pulse 72/min, weak filling and tension.
A. Epidemic typhys
B. Hemorrhagic fever
C. Typhoid fever
D. * Viral meningoencephalitis
E. Leptospirosis
329. Patient D., 53 years, have been examined by doctor on the 2nd day of illness.
Fell ill suddenly, temperature 40 °Ñ, sharp headache, repeated vomiting,
photophobia. The common state is severe, consciousness is darkened,
expressed rigidity of cervical muscles, positive Kernig’s symptom. Pulse 110/min,
A. weak. BP 60/30
Hemorrhagic mm/hg. What is your diagnosis?
stroke
B. Leptospirosis
C. Flu, toxic form
D. * Viral meningoencephalitis
E. Marburg hemorrhagic fever
330. Patient V., appealed to the surgeon with expressed edema of left half of neck.
During examination a doctor observe a carbuncle on the lateral surface of neck
and a few small bubbles near it, filled with a rather yellow liquid. Submandibular
and anterior cervical lymph nodes are enlarged and painful. What diseases is the
A. most credible?
Carbuncle of neck
B. * Anthrax
C. Plague
D. Diphtheria
E. Skin leishmaniosis
331. Patient Í., 21 year, entered hospital on the 7th day of disease with complaints
about the expressed weakness, pains in muscles and joints, head pain, nausea.
The disease began with irritation on the throat, chill, general weakness. After 2
days the state became worse, temperature increased to 39-40 °Ñ. Objectively:
patient adynamic, consciousness is stored. Temperature of body 37,5 °Ñ, the
state as severe, pale skin, cyanosis of lips and nail phalanxes, the scopes of
heart are extended to the left on 1,5 cm, tones deaf, pulse 130/min, weak filling,
BP 80/40 mm/hg. Ring like erythema at forearm. Vesicular breathing. Tongue is
A. Epidemic typhus
B. * Viral meningoencephalitis
C. Leptospirosis
D. Hemorrhagic fever
E. Typhoid fever
332. Patient, 23 y.o., suddenly experienced intensive headacke, nausea, pain in a
neck and lumbar area. Temperature of body 39,2 °C. Expressed meningeal
symptoms. Light, tactile, pain hyperesthesia. Blood: leucocytes – 17 000 /l, ESR-
A. 29 mm/hour. What method of investigation is most informative?
Echoencephalography
B. Transcranial dopplerography
C. Electroencephalography
D. * Lumbar puncture
E. Computer tomography
333. Sick 15 year old, became ill 3 days ago. Disturbed| headache, temperature of
37,4 °C. During next days: headeache increased, a temperature had rise up to
38,3 °C, repeated vomiting. Objectively: a skin is pale, without rash. Pulse
86/min satisfactory filling. Meningeal signs positive. It is needed to conduct for
A. the selection
Endermic testof exciter.
B. Bioassay on mice
C. * Bacteriological examination of CSF
D. Widal test
E. Cultivation on bilious clear soup
334. Sick, 54 years old, hospitalized in an infectious department in a severy
condition. Complaint about the expressed headache, mainly in frontal and
temporal areas, superciliary arcs, origin of vomit on height of pain, pain at
motions by eyeballs, in muscles and joints. Objectively: a patient is excited,
temperature of body 39 °C, BP?100/60 mm Hg. Bradycardia changed by
tachycardia. There were tonic cramps. Doubtful meningeal signs. From
A. Aspirin, analgin, d, dimedrol
B. Lasix, analgin, ampicillin
C. Mannitol, acetophene, prednisolone
D. * Mannitol, lasix, prednisolone, euphylin, suprastin
E. Veroshpiron, euphylin, prednisolone, dimedrol
335. Worker, 22 y.o., became ill sharply: t 39°C, severe pain of head, frequent vomits.
Objectively: the condition is severe, psychomotor excitation, moans due to the
great pain of head, expressed rigidity of neck muscles. Sharply positive
symptoms of Brudzinsky’s and Kernig’s, general hyperesthesia. What’s
A. necessary tomography
Computer to do for clarification of diagnosis?
of cerebrum
B. * General blood analysis and liquor analysis

C. X-ray of skull in two projections


D. Analysis of the excrement and urine for a pathological flora
E. General blood analysis and blood sterility
336. Patient B., 38 years old, was treated from myocarditis in a cardiologic
department. There are violations of cardiac conductivity on the type of
atrioventrikular blockade, tachicardia. The temperature of body is subfebril.
Arthritis of right knee-joint. Works as a forest ranger, likes to gather mushrooms
A. and berries. What is treatment?
Benzylpenicillin
B. Nonsteroid ant inflammatory drugs
C. Glucocorticoids
D. Physical therapy
E. * All above enumerated
337. Young woman, a milkmaid, was admitted to the infectious department. Became
ill 3 days ago. An itching node appeared on the neck. She continued to work, her
state became worse on the eve of hospitalization, body temperature – 38,1?C,
edema at neck and upper part of thorax, with serous-hemorrhagic excretions,
not painful. No animal disease was registered during the last year in diary
A. Tularemia
B. Skin leishmaniosis
C. * Anthrax
D. Plague, skin-bubonic form
E. Furuncle
338. ?The citizen of rural region is hospitalized with complaints about the whole-body
cramps, mouth-opening impossibility. His general condition is poor: temperature
39,5 degrees C, increased tonus of the masseter muscle, wrinkled forehead,
eyebrows and wings of nose, eyes are squinted, muscles of the back of head are
rigid. There are no defecation and diuresis. The clonic and tonic cramps of trunk
A. Rabies
B. Epilepsy
C. * Tetanus
D. Leptospirosis
E. Sepsis
339. The citizen of rural region, 70 years old, is hospitalized in 2 weeks after the
trauma of right brush with complaints about pain in the wound area. His
condition is poor, temperature 39,5 °C, rigidity of the muscles of the back of
head. His mouth opens fully. Tone of muscles is ordinary. What inspections are
A. necessary
General for theof
analysis clarification
blood of the diagnosis?
B. Biochemical analysis of blood
C. * Sowing of the spinal fluid for the selection of tetanus
D. Detection of C. tetani from defecating
E. Determination of antibodies
340. Patient of 15 years, after 3 days of fever and moderate symptoms of intoxication
suddenly felt the signs of peripheral paralysis of the left leg, which were not
conducted with the change of sensitiveness. What disease it might be?
A. * Poliomyelitis
B. Brucellosis
C. Pseudo tuberculosis
D. Tetanus
E. Rabies

341. The patient C., 18 years, is hospitalized to the infectious separation department
with suspicion on a facial tetanus. What diseases should be excluded during the
A. differential
Inflammation diagnostic?
of the lower jaw joint
B. Cranium cerebral trauma
C. Appearance of the wisdom tooth
D. Rabies
E. * All answers are correct
342. Boy of 13 years during a walk was bitten by an unknown dog in a park. The
parents of child appealed to the trauma department with documentary
confirmed complete course of the planned inoculations against the tetanus.
A. What should
Surgical perform
treatment of the doctor for the prophylaxis of tetanus in this specific
wound
B. * Surgical treatment of wound + introduction of 0,5 ml an tetanus anatoxin.
C. Surgical treatment of wound + introduction of 250 ME AT human immunoprotein
D. Surgical treatment of wound + introduction of 3000 ME AT serum
E. Surgical treatment of wound + introduction of 0,5 ml tetanus anatoxin and 250
ME AT human immunoprotein
343. Builder of 37 years came to the trauma department with the chopped wound of
shin; he was twice vaccinated in the past and revaccinated according to the plan
vaccination list 11 years ago. What will be the specific prophylactic of the
A. * tetanus
To injectin thistetanus
anti case? anatoxin + AT serum
B. To inject AT immunoglobulin
C. To inject diphtheria anatoxin
D. To inject DTP vaccine
E. To inject AT anatoxin
344. Patient B., 28 years old accidently got the burn of 2nd degree. He wasn’t
vaccinated against the tetanus before. What should be done for the urgent
A. prophylactic of the tetanus?
Tetanus anatoxin
B. DT vaccine
C. * Tetanus anatoxin + anti tetanus serum
D. Tetanus anatoxin + anti tetanus serum + tetanus immunoglobulin
E. Anti tetanus serum + tetanus immunoglobulin
345. Patient B., 45 years old, came to the hospital on the 6th day of illness with
complaints about headache, tingling, insomnia, depressive mood, pain and
itching at the place of the dog bite 2 months ago. During the examination:
general condition is quite moderate, temperature is 37,6 C, pulse 88 / min, BP
110/60 mm of Hg., apathy, on his face there is an expressions of fear. The skin is
pale, on right lower extremity, in the place of dog bite there is a red scar, around
A. Biochemical blood test
B. * Research of imprints of cornea – to find the presence of antigens of rhabdovirus
C. Research of imprints of mouse – to find the presence of antigens of rhabdovirus
D. Research of imprints of nose – to find the presence of antigens of rhabdovirus
E. Look for Babesh-Negry‘s bodes
346. A teenager was bitten by a neighbor’s dog. This dog is not instilled; it’s on a
leash and bitted the boy after provocation in the area of leg. What should be
A. done for theofprophylaxis
Elimination dog of hydrophobia?
B. Inoculations on vital testimonies
C. Conditional course of inoculations
D. * Treatment of wound
E. All answers are correct

347. A teenager was bitten by a neighbor’s dog. This dog is not instilled; it’s on a
leash and bitted the boy after provocation in the area of hand. What should be
A. done for theofprophylaxis
Elimination dog of hydrophobia?
B. Inoculations on vital testimonies
C. * Conditional course of inoculations
D. Surgical treatment of the wound
E. All answers are correct
348. Woman of 33 years was attacked and bitten with a bat in the area of hand.
During the laboratory research the diagnosis of hydrophobia was confirmed.
What should be done for the urgent prophylactic of the hydrophobia in this case?
A. Gamma-globulin and 18 doses of vaccin
B. 12 doses of vaccin
C. * Gamma-globulin and 6 doses of vaccin
D. Gamma-globulin and 21 dose of vaccin
E. 2 doses of vaccin
349. Student 27 years, ill gradually: appeared hyperemia, was swollen and itch of skin
in area of scars on the right forearm (three months ago was bitten by a dog-fox).
On the second day the temperature of body raised to 38 °C, the symptoms of
intoxication, hydrophobia, photophobia and sense of fear appeared. Periodically
A. * there were paroxysms of cramps. What is the preliminary diagnosis?
Rabies
B. Tetanus
C. Sausage-poisoning
D. Meningococcemia
E. Viral encephalitis
350. The sick C., was hospitalized in an infectious hospital with complaints about
thirst, impossibility to drink water. 1,5 months ago a patient was bitten by a dog
in a right caviar. In area of bite feels burning, itch and heaving up pains. What
A. * prognosis
A of this disease?
disease always ends with death
B. Convalescence is possible in 15 % of patients by using of desintoxication
C. therapy
Specific immune globulin conduces to convalescence
D. Vaccine conduces to convalescence
E. Three days vaccination conduced to convalescence
351. Student of 22 years, became ill gradually: hyperemia and erythema appeared in
the area of right forearm, which was gradually increased to 10 cm in a diameter.
Erythema has a center and raised bright red edges. The temperature of body is
subfebrile, general state satisfactory. 5 days prior to beginning of illness gather
A. mushrooms in the forest. What is our preliminary diagnosis?
Erysipelas
B. * Lime-borreliosis
C. Phlegmon
D. Erysipelotrix
E. Herpetic infection
352. Student of 22 years, became ill gradually: hyperemia and erythema appeared in
the area of right forearm, which was gradually increased to 10 cm in a diameter.
Erythema has a center and raised bright red edges. The temperature of body is
subfebrile, general state satisfactory. 5 days prior to beginning of illness gather
A. * mushrooms in the forest. What is etiotropic therapy?
Tetracyclinum
B. Metronidazolum
C. Normal immunoglobulin
D. Glucocorticoids

E. Enterosorbents
353. A man 28 years old was appealed to the policlinic. The day before he find a tick
into the surface of his skin. He was in-field 2 days ago. He feels satisfactory.
A. What measures
Treatment of theofplace
Limeofdisease
bite prophylaxis?
B. * Urgent antibiotic therapy
C. Using of specific immunoglobulini
D. Enterosorbents
E. Vitamins
354. A., 25 years, got back from Far East. The temperature of body rose up to 39,9 °Ñ
suddenly, pain appeared, rash as a “jacket”, single hemorrhages on a skin. In 3
days weakness, thirst, grew together with the decline of fever, diminished to 300
ml diuresis, the arteriotony went down. Considerably expressed Pasternatsky
A. * Hemorrhagic
symptom. What is most
fever credible
with a kidney diagnosis?
syndrome
B. Congo-Crimea hemorrhagic fever
C. Epidemic typhus
D. Glomerulonephritis
E. Leptospirosis
355. A patient, 39 years old, became ill sharply 3 days ago after return from Jamaica.
Complains on a high temperature, severe headache, nausea, vomit by “coffee-
brown”, pain in the muscles and joints. Objectively: temperature 39,9 °C, face
swelling, red, conjunctivitis, photophobia. Rashes. Scleritis, and skin are yellow,
petachial rash, acrocyanosis. A liver is megascopic to 3 ñm, painful. Tachycardia,
A. * Yellow fever
B. Viral hepatitis B
C. Malaria
D. Leptospirosis
E. Hemorragic fever with a kidney syndrome
356. A woman, a resident of Nigeria, became ill sharply. Appeared a chill, repeated
vomiting, body temperature had risen to 39,7 °C. A patient is excited.
Hyperemia and puffiness was marked of face, expressed sclerite, hyperemia of
mycoses mouth cavity. Pulse 130 per 1 min. The icterus of skin and sclera
appeared on a 3th day, hemorrhages on a skin. A liver and spleen is enlarged.
After the brief improvement in the 5th day of illness the state became worse
sharply. A temperature again rose to 39,9 °C. Raves. An icterus grew sharply.
A. Viral hepatitis A
B. Leptospirosis
C. * Yellow fever
D. Crimean fever
E. Malaria
357. A patient, habitant of Nigeria, grumbles about the increase of temperature,
general weakness. Gradually a temperature rose to 38,9 °Ñ, pain of head
increased, pains appeared in muscles, nausea, conjunctivitis. On a 3th day
necrotizing ulcer pharyngitis developed. Ulcers appeared on a soft palatine
tonsils. The groups of lymphatic nodes of neck were multiplied. The state got
A. Flu
B. Typhoid fever
C. * Hemorrhagic fever
D. Quinsy
E. Spotted fever

358. Patient T., 22 years old, was hospitalized in the infectious department with
complaints on a chill, fever, severe headache, pain in the gastrocnemius
muscles. The state is severe, hyperemia of face, icterus of skin and scleras, liver
and spleen are enlarged, olygouria. BP is 60 and 20 mm Hg, pulse – 120 per a
A. min. What preparations
Hormones, musttherapy,
dethintoxikation be entered?
antibacterial preparations
B. Diuretic, dethintoxikation therapy, antibacterial preparations
C. Sorbents, diuretic, dethintoxikation therapy
D. * Hormones, diuretic, dethintoxikation therapy, antibacterial preparations
E. Hormones, diuretic, antibacterial preparations
359. ?What disease is this? A painful lesion with a bright red, edematous, indurate
appearance and an advancing, raised border that is sharply demarcated from
A. the adjacent normal skin. Fever is a feature.
Phlegmona
B. Anthrax
C. * Erysipelas
D. Thrombophlebitis
E. Eczema
360. Patient A., 57 years old, farmer, came to the doctor on the 3th day of illness with
complaints on headache, pain in the gastrocnemius muscles, fever, icterus of
skin and scleras, dark urine and decries level of urine, temperature of body –
38,2 °C, petehial rash on overhead part of thorax, hepatosplenomegalia. What
A. preparations
Salts must be entered?
solutions
B. Transfusion of fresh-frozen plasma
C. * Antibiotics
D. Hyperbaric oxygenation
E. Hemosorbcion, sympathomimetics
361. Patient K., 43 years old, miner, on the 4th day of disease complaints on the
weakness, headache, pain in the gastrocnemius muscles, fever, icterus of skin
and scleras, dark urine, temperature of body – 38,2 °C, hepatosplenomegalia.
What preparations must be entered? There was the nose-bleed. Diuresis – 200
A. * Antibiotics
ml. What preparations must be entered?
B. Holynomimetics
C. Spasmolytics
D. Miorelaxants
E. Vitamins of group B
362. Patient R., 48 years old, admitted to the clinic on the 4-nd day of disease with
complains of fever, headache, chill, burning pain and redness of left leg; Ò of the
body is 38,9 C. It appears as a red, swollen, warm, hardened and bullae, and
petechiae. The margin of erythema looks like „tongue of the flame”. During
palpation skin has cake-like consistency. Which is the most credible diagnosis?
A. Eritematozis
B. * Eritematozo-bullezis
C. Hemorrhagic
D. Eritematozo-hemorrhagic
E. Necrosis
363. Patient V., 55 years old, admitted to the clinic on the 5-nd day of disease with
complains of fever, headache, chill, burning pain and redness of left leg; Ò of
body is 39,1 C. It appears as a red, swollen, warm, hardened and bullae with
hemorragic content, and petechiaes. The margin of erythema looks like „tongue
of the flame”. During palpation skin has cake-like consistency. Which is the most
A. Eritematozis

B. Eritematozo-bullezis
C. Hemorrhagic
D. * Eritematozo-hemorrhagic
E. Necrosis
364. A new born child on 10th day of life became worse: temperature - 39.2 °C, no
frequent vomiting, generalized cramps, violations of consciousness, spastic
paresis of left extremities. Month prior to his birth herpes virus was present in
the mother,
A. * Herpetic which she did not treat. What disease is the most possible?
encephalitis
B. Poliomyelitis
C. Subarechnoid hemorrhage
D. Cerebral abscess
E. Violation of blood cerebral circulation
365. A patient 60 years old for 2 days has disturbed severe pain in a right arm. On
3rd day appeared blisters, pouring out as a chain on the skin of shoulder,
forearm and brush. Sensitiveness in the area of pouring out is mionectic. What
A. disease can be diagnosed?
Dermatitis
B. * Herpetic ganglionitis
C. Neck-pectoral rediculitis
D. Psoriasis
E. Allergy
366. A pregnant woman, 27 years (pregnancy ²², 8-10 weeks.), temperature of body
increased. At the inspection on a TORCH-infection antibodies are found Ig M to
the herpes virus, ²² types. What we must recommend to pregnant women?
A. * To cut pregnancy
B. To prolong the supervision
C. Treatment with acyclovir
D. Symptomatic treatment
E. Appoint of alpha-fetoprotein
367. Patient S., 44 years old, appeal to the physician with complains of chills,
increased body temperature till 390 C. After some hours edematic area with
redness appeared on the skin of face. Hyperemia is vivid with sharp borders, on
the periphery presence of shaft, margin of infected area is not regular, similar to
the map picture. In anamnesis: 5 years ago had erysipelas, whit was localization
A. Preview
B. * Repeated
C. Recurrent
D. Continuously-recurrent
E. Chronic
368. Patient T., 61 years old, appeal to the physician with complains of chills,
increased body temperature till 38,5 C. After some hours edematic area with
redness appeared on the skin of face. Hyperemia is vivid with sharp borders, on
the periphery presence of shaft, margin of infected area is not regular, similar to
the map picture. In anamnesis: the inflammatory process of that localization
A. Preview
B. Repeated
C. * Recurrent
D. Continuously-recurrent
E. Chronic

369. A patient, 22 years old, became ill sharply. History showed fever up to 38.2 °C
with headache, repeated vomiting, olfactory and tastes hallucinations. Quickly
got complex of meningeal symptoms, pyramidal paresis. The general epileptic
attack and comatose state also developed. Neurolymph is with mixed
A. lymphocytosis,
Brain abscess cytochrome, single red corpuscles. What is previous diagnosis?
B. Subdural empyema
C. * Herpetic encephalitis
D. Tumor of brain
E. Encephalopathy
370. A boy 11 years old, complaints about sickness at mastication, increasing of
temperature to 37.1 °C, enlarge of parotid salivary glands . At the age of 8 years
carried a paraflu infection. Objectively: in the region of right parotid salivary
gland tubular sickly at palpation, a skin above it is not changed. A pharynx is
A. * Cytomegaloviral
moderate hyperemic, tonsils are not coated. What is your previous diagnosis?
syaloadenitis
B. Lymphadenitis
C. Parotitis
D. Infectious mononucleosis
E. Mamps
371. A woman 65 years old had the disease beginning sharply from increase of
temperature to 39.0 °C, weakness, and pain in the left part of thorax that
increased with breathing motions. On 3rd day of disease appeared after motion
of rib on the left on a hyperemic background. Together with sick a grandchild
A. lives 4 years. What measures of prophylaxis of disease need to be adopted?
Vaccination
B. Final disinfection
C. Reception of specific immunoprotein
D. * Isolation of patient
E. Acyclovir administration
372. The patient, 58 years old, was hospitalised in the infectious department with
complaints of pain in the left half of thorax, fever. At a review: temperature of
body 37.5 °C, in XI-XII intercostal area of skin are groups of the vesicular rash.
A. Preparation of choice for treatment of this patient is:
Suprastin
B. Prednisolone
C. Biseptolum-480
D. Semavin
E. * Laferon
373. A female M., 24 years old came to the doctor with long-lasting fever, night
sweat. The weight reduced by 7 kg during the three months. Objective
examination found an increasing of all lymph nodes, hepatolienal syndrome. In
the blood:
A. * HIV Leucocyte – 2,2x109. What disease should be suspected?
infection
B. Lymphogranulomatosis
C. Tuberculosis
D. Infectious mononucleosis
E. Sepsis
374. Patient A., 35 years old, came to the clinic on the third day of disease with
complaints of drowsiness, sweat, headache, fever up to 38,5 °C. Reviewing
physician found sore throat, splenomegaly, enlarged lymph nodes. Liver not
enlarged. Blood analysis – leukocytosis, lymphomonocytosis. What are the
treatment for
A. * Antibiotic, thisinterferon,
drugs patient? hepatoprotectors
B. Antihistamin, antiherpetic preparations hepatoprotectors

C. Antibiotic, hepatoprotector, antihistamine


D. Antibiotics, antihistamine, antiherpatic drugs
E. Vitamins, antibiotics, drugs interferon
375. Patient 18 years, has complaints for headache, general weakness, increase of
temperatures to 37,5–38,2 °C during 6 days, pharyngalgia. Objectively: palpated
all groups of lymphonoduses, 1-3 cm in a diameter, elastic, not soldered
between itself. A liver is enlarged on 3 sm, spleen – on 1 sm. In a blood is
A. leycocytosis, plasmatic cells – 15 %. What group of herpes does the exciter of
Alpha
B. Beta
C. * Gamma
D. Delta
E. Teta
376. Sick men, 24 years old, disturbs a pharyngalgia, general weakness. Objectively:
temperature 38,0 °C, pulse 96 shots per min., tonsils are enlarged in sizes,
coated. Beck neck and submandibular lymphatic nodes are palpated enlarged,
the edge of spleen is under the costal arc to 1 sm. Point rashes on the skin of
A. trunk. What
Scarlet feverdisease does it follow to think for?
B. Acute respiratory disease
C. Follicle quinsy
D. Typhoid
E. * Infectious mononucleosis
377. Patient C., 18 years old, appealed in a policlinic with complaints about a
moderate pharyngalgia, headache, general weakness, T 38,9 °C. He has been ill
three days. Objectively: enlarged back neck, and inguinal lymphatic nodes,
hepatosplenomegaly, subicterus scleroticas and skin, changes in the throat like
to angina. At the common analysis of blood – lymphomonocytosis. What are
A. Antibiotics, hepar protective preparations, antihistaminic
B. Antiherpetic preparations, hepar protective preparations, antihistaminic
C. * Antibiotics, preparations of interferon, hepar protective preparations
D. Antibiotics, antiherpetic preparations , antihistaminic
E. Antibiotics, preparations of interferon, vitamins
378. At a patient, 17 years: tonsillitis, temperature 38,2 °C, generalized
lymphadenopathy (the first multiplied neck lymphatic nodes which are located
along m. sternocleidomastoideus), hepatospleenomegaly. What exciter does
A. cause
Herpesthis disease?
virus I type
B. Herpes virus ²² type
C. Herpes virus ²²² type
D. * Herpes virus ²V type
E. Herpes virus V type
379. Sick man, 24 years, is disturbs a pharyngalgia, general weakness. Objectively:
temperature – 38 °C. Pulse 96 shots per min, tonsils are enlarged in sizes,
coated. Palpated enlarged back neck and submandibular lymphatic nodes, a
spleen is + 1 sm. What methods of diagnostics is possible to confirm a diagnosis
A. by?
Reaction of Paul-Bynnel, Right-Heddlson, Goffa-Bauer
B. * Reaction of Paul-Bynnel, Lovrik-Volner, Goffa-Bauer
C. Reaction of Paul-Bynnel, Right-Heddlson, Lovrik-Volner
D. Reaction of Paul-Bynnel, Goffa-Bauer
E. Reaction of Paul-Bynnel, Goffa-Bauer, Right-Heddlson, Lovrik-Volner

380. The boy 12 years old, with catarrhal phenomena seen an increase of all the
lymph nodes, scleratis, hyperplasia of tonsils, white layers on them as islands,
existing single maculo-papular rash, increased liver and spleen size. What
A. additionalofresearch
Reaction passive should be appoint?
heamaglutination(RPG) with influenza viruses
B. Ultrasound of the abdomen
C. Puncture of the lymph nodes, with following microscopy
D. Inoculation of blood
E. * Blood test for atypical mononucleares
381. At a 36 years old sick person, a rash appeared on a skin 4 days ago,
accompanied by itching. Swelling of stomach. The pain Disturbed in right
subcostal area had constipation. Day prior to it he ate the smoked meat.
Atypical reactions appeared after the use of tomatoes, strawberry, chocolate.
Objectively: on the skin of person, trunk, extremities are rashes. Level of general
A. * Pseudoallergy
B. Idiosyncrasy
C. Food allergy
D. Herpetic infection
E. Chronic hives
382. A 3 years old child complains about: cough, temperature of body 37.9 °C.
Conjunctiva is hyperemic. On mucous of cheeks there are points of hyperemia.
Weaken breathing in the lungs. What is the most possible diagnosis?
A. Scarlet fever
B. Rubella
C. * Measles
D. Herpetic infection
E. Flu
383. A 24 y/o woman was seen by a doctor on the 2d day of illness with complaints
about a fever up-to 38,1 °C, painful swallowing, malaise. Point rash on
hyperemic background was found on the skin of trunk and extremities,
especially in natural folds,. The nasolabial triangle was white with no rash.
Enantema on a soft palate, and purulent exudates on tonsils were observed. A
A. * Scarlet fever
B. Rubella
C. Measles
D. Enteroviral infection
E. Flu
384. A 24 y/o patient was seen by a doctor on the 2d day of illness with complaints
about a fever up-to 38 °C, headache, and malaise. Vesicular rash with red hallow
was found on the skin of hairy part of head, trunk and extremities. Also some
vesicular elements were found on the mucous membrane of pharynx. Lymph
A. nodes were
Scarlet fevernot palpable. What is the most probable diagnosis?
B. Rubella
C. Measles
D. * Chicken-pox
E. Herpetic infection
385. Patient A., complains of redness of skin and edema on the right cheek. During a
review: Ò 38,7 C, enlarged and painful lymphatic submandibular nodes of right
side, border between red and normal skin is sharp, present of swallowing with
A. dark content,
Herpetic palpation is painful. What is your previous diagnosis?
infection
B. Anthrax, skin form

C. * Erysipelas
D. Chicken pox
E. Phlegmone of cheek
386. Patient S., 44 years old, appeal to the physician with complains of chills,
increased body temperature till 39 C. After some hours edematic area with
redness appeared on the skin of face. Hyperemia is vivid with sharp borders, on
the periphery presence of shaft, margin of infected area is not regular, similar to
A. the map picture.
Stephansky’s Which symptom does prove the most possible diagnosis?
symptom
B. * Andrec’s symptom
C. Simonovsky’s symptom
D. Brunner’s symptom
E. Filatov’s symptom
387. Patient S., 44 years old, a worker of pig-breeding farm, asks for help to physician
with complains of pain, excessive edema and flexion contracture intraphalangeal
joints of middle and index finger of right hand (between ² and ²² phalanges).
A. There are noofchanges
Phlegmone bone of bones on X-ray. What is your diagnosis?
B. * Erysipelotrix
C. Reactive arthritis
D. Erysipelas
E. Rheumatoid arthritis
388. What is the base prophylactic measures of erysipelas relapse?
A. Administration of antistreptococcus immunoglobulin
B. Usage of small dosage of hormones
C. * Bicillin prophylaxis once in a month
D. Usage of vaccine
E. Prevention of skin trauma and angina development
389. Patient Ê., 43 years old, hunter. The onset was acute: appeared sharp pain in
right inguinal area, which lead to extortion position of hand. Temperature raised
till 39,5 rC. Next day he calls the doctor who put the diagnosis “Lymphadenitis”
and directs the patient by ambulance to surgical department. Surgeon during
examination revealed woody-like tightly conglomerate of lymphatic nodes and
A. * Plague, bubonic form
B. Sepsis, purulent lymphadenitis
C. Purulent lymphadenitis
D. Òhrombosis of a. femoralis
E. Òularemia, bubonic form
390. Patient, 37 years old hospitalized in infections department on the 3rd day from
the beginning of the disease in very severe condition. There are complains of
high fever and sharp general weakness, strong pain in inguinal area. Objectively:
consciousness is darkened, Ò – 41 C, in right inguinal area the group of
enlarged, connected lymphatic nodes, not moveable, tightly, unclear borders,
very painful, redness of skin above them. After one day appeared pain in right
A. * Plague, secondary-lung form
B. Anthrax, generalized form
C. Plague, primary-lung form
D. Lung tuberculosis
E. Òularemia, lung form

391. Patient N., 35 years old, admitted at 15th day of disease with subfebrile
temperature. On the right side of the neck palpates lymphatic node looks like
chicken egg in size, moderate pain, not connected with surrounding tissues. On
the back side of the neck dry spot of dark-brown color. What is the most possible
A. diagnosis?
Plague, bubonic form
B. Anthrax
C. Purulent lymphadenitis
D. Mononucleosis
E. * Òularemia, skin-bubonic form
392. Citizen Ê., goes to country with unfavorable conditions about plague. Provide
necessary measures of specific prophylaxis.
A. Human immunoglobulin
B. ²nterferon
C. * Dry alive vaccine
D. Bacteriophags
E. Alive measles vaccine
393. The student, who arrived on vacation from Sierra Leone, was taken to hospital in
connection with the febrile illness. Suspected Lassa fever. What kind of
laboratory indicators are characteristic for this disease?
A. The decline in serum indicators KFK and LDG
B. ESR greater after 60 mm/hour
C. Neutrophiil in liquor
D. Elevated levels of alkaline phosphatase in the serum
E. * High proteinuria
394. A 30-year-old resident of Peru brought to the hospital at the 4th day of illness,
there was vomiting with blood, nose bleeding, icteric skin, petehii. In urine
revealed erythrocytes protein. Volume of urine decreased. What is the
preliminary
A. * Yellow fever diagnosis?
B. Dengue Fever
C. Malaria
D. Lassa fever
E. Ebola fever
395. In ambulance delivered a girl 14 years, which became ill 6 days ago from
appearance of weakness, increase body temperature to 39,3 °C, enanthema
appeared on the skin and mucouses and had hemorragic character, icterus,
nasal bleeding, increases of liver and spleen. 2 weeks ago got back from a
A. * Yellow
festivalfever
in Brazil. What infectious pathology should be eliminated first?
B. Q-fever
C. Dengue fever
D. Malaria
E. Viral hepatitis
396. At a boy 10 years, which lives in Transcarpathia, on the 4th day of fever
appeared painless hemorrhagic petechial rushes, sometimes in the form of red
and purple stripes with overwhelming localization on a neck, thorax, in the
auxiliary, above collar-bones. On hyperemic mucus of oro-pharing point
hemorrhages, bleeding from a nose. In lungs was hard breathing, tones of the
heart deaf, bradicardia, swelling and stomach-ache, enlarged a liver and spleen.
Oliguria, proteinuria, hematuria, cylindruria. In blood neutrophilic leukocytosis,
A. Meningococcal infection
B. * Hemorragic fever

C. Rheumatism
D. Leptospirosis
E. Thrombocytopenic purpura
397. Patient, 37 years old hospitalized in infections department on 3rd day from the
beginning of the disease in very severe condition. Complains of high fever and
sharp general weakness, strong pain in inguinal area. Objectively: consciousness
is darkened, Ò – 41 C, in right inguinal area the group of enlarged, connected
lymphatic nodes, not moveable, tightly, unclear borders, very painful, redness of
skin above them. After one day appeared pain in right side of thoracic cavity,
A. * Plague, secondary-lung form
B. Anthrax, generalized form
C. Plague, primary-lung form
D. Lung tuberculosis
E. Òularemia, lung form
398. Patient S, 52 years old, cleaner, came to outpatient department with complains
of pain sensations in area of right forearm, where appeared abscess, sharply
painful with infiltration, mild edema and hyperemia of surrounding skin. In the
A. center there
Plague, is deep ulcer. Palpation is painful. What is the primary diagnosis?
skin form
B. Anthrax, skin form
C. * Furunculus
D. Òularemia, skin form
E. Sepsis, purulent lymphadenitis
399. 11 Patient Ê., 43 years old, hunter. The onset was acute: appeared sharp pains
in right inguinal area, which lead to extortion position of hand. Temperature
raised till 39,5 rC. Next day he calls the doctor who put the diagnosis
“Lymphadenitis” and directs the patient by ambulance to surgical department.
Surgeon during examination revealed woody-like tightly conglomerate of
A. * Plague, bubonic form
B. Sepsis, purulent lymphadenitis
C. Purulent lymphadenitis
D. Òhrombosis of a. femoralis
E. Òularemia, bubonic form
400. Patient L., 38 years old, complains of significant pain in right inguinal area, takes
extorsion position of extremity. From epidemiologic anamnesis revealed, that he
works as a loader in port. During punctuate microscopy from bubon revealed
bipolar stained gram-negative small sticks. What is the transmitting agent of this
A. disease?
Flies
B. Mosquito
C. * Flea
D. Lice
E. Tick
401. At the patient B., 32 years old, in 1th day of illness, temperature of body are
40 °C, delirium, pulse 140 per 1 min, AP is 80/40 mm Rh, vomiting, petehial rash
on a skin. In the iliac region painful conglomerate of enlarged lymph nodes,
dense and elastic in consistency. In anamnesis – a patient got back from
Vietnam 3 days ago. What is your previous diagnosis?
A. * Plague
B. Rabbit-fever
C. Anthrax
D. Furuncle of iliac area
E. Pseudotuberculosis

402. Patient C., 41 years old, geologist, was during 2 months to in the steppes of the
Astrakhan before illness. Became ill suddenly on a 8th day after return home:
temperature of body – 40,0 °C, headache, vomiting, chill. A temperature grew,
appeared a weakness, dizziness. Cardiac tones are muffled, pulse – 110 per a
min, tongue is “chalky”. On the skin of right shin sharply sickly ulcer 2-3 sm,
covered by a dark scab. In a right inguinal area painful conglomerate of enlarged
A. Rabbit-fever
B. Anthrax
C. Phlegmon of right shin
D. * Plague
E. Spotted fever
403. Patient D., 42 years old, hunter, became ill suddenly: temperature of body
increased to 40,0 °C, chill, sharp headache, vomiting. He was hospitalized in the
1-t day of disease in severe condition: excited, temperature of body 40,0 °C, a
face is hyperemic, eyes brilliant, language is bed. Skin are moist, clean.
Breathing is vesicular, single small-vesicles moist wheezes. At deep inhalation
severe pain in the right half of thorax. BP is 90/60 mm of Hg, pulse – a 120 per a
A. Q-fever
B. Rabbit-fever
C. Spotted fever
D. * Plague
E. White plague
404. In infectious department delivered a patient M., 22 years old, with complaints on
weakness, dizziness, vomiting, insomnia, chills, fever, acute pain in right iliac
region. Second day of disease. During examination: impaired consciousness,
insignificant excitation, weak of cardiac sounds, tachycardia, pulse with weak
filled and tensions. In lungs changes were absent. By palpation, enlarged spleen.
Ordinary sizes liver. In the right iliac region painful conglomerate of enlarged
lymph nodes, dense and elastic in consistency, hyperemic skin above them. It is
A. * Plague, bubonic form
B. Anthrax, generalized form
C. Plague, primary-pulmonary form
D. Tuberculosis of lungs
E. Tularemia, pulmonary form
405. Patient E., 43 years old, hunter, often drank water from springs during hunts on
hare. Became ill sharply in 2 days: chill, high temperature, pharyngalgia at
swallowing. Objectively: mucus of oropharynx not sharply hyperemic, on right
tonsil dense grayish-yellow raid. In the right part of neck enlarged lymph nodes
A. like to eggs, not painful, mobile, temperature – 38 °C. What is previous
Diphtheria
B. Plague
C. * Rabbit-fever
D. Tonsillitis
E. Limphogranulomatosis
406. A boy F., 14 years old, has been hospitalized in infectious department with
complaints on severe headache, temperature increased to 40,2 °C, edema and
pain in the iliac areas. Got back from Russia three days ago, where was on hunts
and eat a meat of hare, together with a father. Objectively: a face is hyperemic,
the iliac lymphonoduss are enlarged, skin above them hyperemic, light, moving
A. Limphogranulomatosis
B. Rabbit-fever
C. Sharp festering lymphadenitis

D. * Plague
E. Anthrax
407. In a boy 12 years old, which got back from Mongolia with parents 5 days ago,
suddenly fever with chill, great headache, muscles pain, deliriums, face – bright
red, injection of sclera, tongue is dry, edematous, covered with white coating. In
the right iliac region painful conglomerate of enlarged lymph nodes, dense and
elastic inbubonic
A. * Plague, consistency,
form hyperemic skin above them. What is previous diagnose?
B. Anthrax, skinning form
C. Rabbit-fever, bubonic form
D. Regional lymphadenitis
E. Phlegmon
408. Patient G., 40 years old, complaints of rising of temperature to 39,0 °C, pain in
muscles, headache, general weakness. Became ill suddenly. The state is severe,
pulse – 120/min, BP 90/50 mm of HG, a hepatomegaly. Painful lymphatic nodes
is palpated in a left inguinal area, edema of soft tissues. In a blood leucocytes
A. * Plague,
18õ10,6,bubonic
ESR – 35 mm/hr. What is previous diagnosis?
form
B. Sepsis
C. Lymphogranulomatosis
D. Acute leucosis
E. Anthrax
409. A Sick 18 years, hospitalized in an infectious department with diagnosis of
cholera, as heavy as lead motion, dehydration of IV degree. What measures are
A. possible
Oral primarily?
rehydration by glucose solutions
B. Tetracycline
C. * Intravenous stream introduction of salt solutions
D. Proceeding in the normal microflora of intestine
E. Intravenous stream introduction of sodium chloride solution
410. In a girl in 14 years old, which got back from India 4 days ago with parents,
suddenly fever appeared, malaise, dizziness, delirium, hallucinations, severe
headache, frequent vomiting, palpitation, shortness of breath, moist cough,
appeared with bloody sputum. Petehial-hemorrhagic rash widespread on skin. In
lights it is hyposthenic from both sides of breathing, paravertebral are small
A. * Streptomicinum
B. Oxytetraciklynum
C. Monomycinum
D. Ampicillin
E. Morphicyklinum
411. A girl 13 years old, is delivered in child's infectious department with complaints
of temperature increased to 40,2 °C, headache, pain and edema in the iliac
region. Family of child got back from Middle Asia two days ago, where prepared
meat of camel. Objectively: the skin of face is hyperemic, dramatic face. In the
left iliac region painful conglomerate of enlarged lymph nodes, dense and elastic
A. * Plague
B. Sharp festering iliac lymphadenitis
C. Reticulosis
D. Rabbit-fever
E. Lymphogranulomatosis

412. State of patient F., 37 years old, was severe. He had complaints of high fever,
general weakness, pain in an inguinal area. Objectively: consciousness, altered
temperature, T – 41,0 °C, in a right inguinal area package from enlarged, painful
conglomerate of lymph nodes, immobile, dense, unclear, skin above nodules
was red. After a days there was pain in the right half of thorax, appeared cough
A. Tuberculosis of lungs
B. Plague, initially-pulmonary form
C. Rabbit-fever, pulmonary form
D. Anthrax, generalized form
E. * Plague, secondary-pulmonary form
413. Severe pain appear at a patient in right iliac region ten hours ago. Temperature
rose to 39,2 °C. Objectively: poor balance, pulse is frequent. . In the right iliac
region painful conglomerate of enlarged lymph nodes, dense and elastic in
consistency, hyperemic skin above them. What diagnosis is most reliable?
A. Anthrax
B. Rabbit-fever
C. * Plague
D. Lymphogranulomatosis
E. Acute festering lymphadenitis
414. Patient H., 48 years old, has been hospitalized with complaints of a rise in
temperature to 40,4 °C, severe headache, shortness of breath, cough, with the
liquid, bloody sputum. He arrived from India a week ago. Breathing in lights is
hyposthenic, are small quantity of moist crackles & crepitation. Tones are weak.
A. What diagnosis
Yellow fever is most reliable?
B. Flu
C. * Plague
D. Chicken pox
E. Cholera
415. Patient J., 23 years old, has been hospitalized in grave condition: altered level of
consciousness, temperature – 41,3 °C, package of enlarged lymph nodes are in a
right iliac area, immobile, dense; very painful and hyperemic skin. After a day
there was pain in the right half of thorax, cough with a bloody sputum. What is
A. previous diagnosis?
Anthrax. Pulmonary form
B. Plague. Initially-pulmonary form
C. Rabbit-fever. Pulmonary form
D. * Plague. Secondary-pulmonary form
E. Tuberculosis of lungs
416. Patient K., 41 years old, feels pain in a left iliac area. At the inspection:
temperature – 39,0 °C, in a left iliac area conglomerate of lymph nodules, in a
diameter 7 cm, dense, painful, immobile, skin above him hot by touch, glitters. A
previous diagnosis is a plague. What etiotropic preparation we must be given to
the patient?
A. * Streptomicinum
B. Penicillin
C. Cefazolin
D. Lincomicinum
E. Ceftriaxonum

417. Sick C., 20 years old, was admitted to the hospital with complaints about the
frequent emptying without stomach-ache, vomiting without nausea, pain in calf
muscle. Objectively: Temperature of body is 36,2 nC. Skin is pale, cold, tongue is
dry, voice is hoarsed. The stomach is pulled is not painful at palpation. Emptying
resemble with “rice-water”. Are there what terms of discharge of patients from
A. * Triple negative results of bacteriological examination of excrements
B. Double negative results of bacteriological examination of excrements
C. Single negative result of bacteriological examination of excrements
D. Single negative results of bacteriological examination of excrements and urine
E. Double negative results of bacteriological examination of excrements and urine
418. Patient C, 17 years old, became ill suddenly. Profuse diarrhea with frequent
vomits without nosier. A patient arrived from south-east Asia. Temperature –
36,1 °C. An abdomen is pulled in, painless. The stool has the appearance of rice-
water. What diagnosis is most reliable?
A. * Cholera
B. Dysentery
C. Salmonellosis
D. Esheryhiosis
E. Rotavirus gastroenteritis

S-ar putea să vă placă și